Old Q&A – Injuries and Toxic Effects

A 57-year-old woman with coronary artery disease associated with hyperlipidemia comes to the clinic for a scheduled follow-up appointment. She saw you for the first time approximately 1 month ago to establish care. In the interim, she was started on hydrochlorothiazide for elevated blood pressure (confirmed on a repeat nurses visit) and on simvastatin for a fasting LDL of 190. She has a remote history of alcoholism, but denies any alcohol intake over the past 10 years. Today, she complains of mild, generalized weakness and states that her shoulders and thighs are “achy”. She denies rhinorrhea, fevers, chills, nausea, vomiting, or diarrhea. While she does not complain of any dysuria, she states that her urine has been very dark for the past few days. She denies abdominal or flank pain. Laboratory studies show a mildly elevated white blood cell count, a normal hematocrit, and normal electrolytes. Her AST (or SGOT) is 415 and her ALT is 25. Bilirubin and alkaline phosphatase are within normal limits. The most appropriate next step in evaluation is to

  A. determine creatinine kinase level
  B. obtain an erythrocyte sedimentation rate
  C. order a GGT level and a serum alcohol level
  D. send Hepatitis A, B, and C serologies
  E. send her for a right upper quadrant ultrasound
Explanation:

The correct answer is A. AST is less specific for liver than ALT. AST is found in multiple organs and will be elevated with any muscle injury. In fact, before the advent of assays for the MB fraction or troponin, AST was used to assess for myocardial infarction. The patient was recently placed on simvastatin (an HMG Co-A reductase inhibitor). While these cholesterol-lowering drugs are generally benign, myositis is a complication that the prescribing physician must be aware of. The patient’s presentation of fatigue and muscle aches fits the diagnosis of myositis. In addition, an elevated AST with an otherwise completely normal liver panel should heighten suspicion that the AST is not coming from the liver. The patient’s dark urine is classic for myoglobinuria. The patient should be treated with intravenous fluids to maintain renal perfusion, therefore avoiding renal tubular injury from the myoglobin. Depending on the level of the creatinine kinase, alkalinizing the urine may also help protect the kidney from injury in this setting. There is no definitive treatment for the myositis itself. Typically, the myositis resolves after the offending agent (simvastatin in this case) is discontinued.

An erythrocyte sedimentation rate (choice B) is incorrect. While an erythrocyte sedimentation rate may be useful as a sensitive marker of inflammation, it is not specific for any disease process. As this case illustrates, recently prescribed medicines should always be considered at the top of your differential diagnosis as the etiology of a new disease process. The erythrocyte sedimentation rate would not help make the diagnosis, nor would the result change management.

An elevated AST to ALT ratio may be suggestive of alcoholic liver injury. This is thought to be due to the fact that ethanol decreases ALT synthesis. In addition, ethanol is thought to cause mitochondrial damage in the liver, where AST lives. However, the ratio of AST to ALT in alcoholic hepatitis is more frequently closer to 2:1. The extremely high ratio in this case (again, with a normal ALT) points to an extrahepatic process. An elevated GGT can reflect alcoholic liver damage, but again, this is unlikely in the face of completely normal bilirubin and alkaline phosphatase. Therefore, ordering a GGT level and a serum alcohol level (choice C) is not correct.

Hepatitis A, B, and C serologies (choice D) are incorrect because the viral hepatitides should not cause an isolated level in AST.

A right upper quadrant ultrasound (choice E) is incorrect mainly for the same reasons as elucidated above. A right upper quadrant ultrasound is useful for evaluating suspected structural disease. Typically, structural disease of the liver is suspected when there are clues of hepatic obstruction. Elevated bilirubin and alkaline phosphatase are typically elevated in hepatic obstruction, and both are normal in this case.

A 20-year-old man comes to your office with a dog bite to his left thigh received after he and a friend taunted a neighbor’s dog. He reports that the bite occurred about 36 hours ago and only came to your office after coworkers informed him that dog bites frequently become infected. His temperature is 37.0 C (98.6 F), blood pressure is 110/70 mm Hg, pulse is 63/min, and respirations are 13/min. On physical examination you notice a shallow abrasion on his left thigh, which is mildly tender. There is no surrounding edema or erythema. The most appropriate management of this patient is to

  A. administer a rabies vaccination
  B. prescribe amoxicillin-clavulanate
  C. prescribe clindamycin
  D. prescribe penicillin
  E. provide local wound care without antibiotic therapy
Explanation:

The correct answer is E. Only 5-20% of dog bites become infected. This patient does not seem to have any systemic signs of infection and his wound doesn’t appear to be infected. Puncture wounds become infected more often than abrasion and this patient doesn’t have any skin punctures. Local wound care including debridement, cleaning, and irrigation are essential. X-rays can be taken if fracture is suspected but a fracture would be unlikely in a superficial thigh wound.

Rabies vaccination (choice A) is not indicated now. Vaccination for rabies should be considered only when you have a strong suspicion of rabies. Rabies is always fatal so treatment is essential if the disease is seriously being considered (such as a bat, skunk, raccoon, fox, bat, etc.). In this case, the dog was a household pet, which probably indicates that it was vaccinated in the past. Also, the dog did not seem rabid (the dog bite occurred only after being provoked). If the dog spontaneously bit the patient, suspicion might be higher. If rabies is suspected, an attempt to contact the dog owner is a reasonable first step to determine the pet’s vaccination status. Local wound care remains important.

Antibiotics would be indicated for patients with signs of local or systemic infection. Hand bites should also be treated with antibiotics since infection can be devastating to a patient. Amoxicillin-clavulanic acid (choice B) and clindamycin (choice C) would both be acceptable antibiotics since they have broad spectrums covering likely infectious organisms such as Pasteurella multocida, Staphylococcus, Streptococci, and anaerobes.

Penicillin (choice D) is a well studied choice for prophylaxis of hand bites but concerns about narrow spectrum of activity have caused many physicians to use alternative antibiotics.

http://www.usmlestep.com

http://health.groups.yahoo.com/group/usmlestep3

Visit us to get more STEP 3 Materials

 

A 32-year-old man is admitted to the hospital following a motor vehicle accident. He was driving a car at 70 miles per hour and was involved in a head-on collision. He was wearing a seat belt at the time of the accident. Initial evaluation in the emergency department by means of physical examination, cervical spine, and chest and pelvic x-rays did not reveal any abnormalities. A complete blood count and metabolic panel were within normal limits. Urine analysis does not show evidence of blood or RBCs. He is admitted to the hospital because of suspected contusion to the brain and because of the death of the passenger sitting in the front seat in the same accident. Twelve hours after admission, he complains of severe abdominal pain. His vital signs are within normal limits except for tachycardia. His abdomen is soft, slightly distended with absent bowel sounds. Repeat laboratory investigation shows a stable hematocrit, leukocyte count of 16,000/mm3, and a serum amylase of 4000 U/L. The most appropriate next step in the management of this patient is

  A. analgesics
  B. CT scan of the abdomen and pelvis with IV contrast
  C. diagnostic peritoneal lavage
  D. observation
  E. ultrasonography
Explanation:

The correct answer is B. Pancreatic injuries should be suspected in blunt abdominal trauma. In motor vehicle accidents associated with seat belt injuries, pancreatic fractures is a known entity. These patients initially may not have any symptoms and signs and may not reveal any abnormality in laboratory investigations. Continued observations sometimes reveal abdominal pain or a mildly tender distended abdomen. Elevation of serum amylase should arouse the suspicion of injury to the pancreas. This pancreatic fracture and disruption of the duct can be classically diagnosed with a CT scan of the abdomen with intravenous contrast.

Analgesics (choice A) are not indicated without an established diagnosis in blunt abdominal trauma.

Diagnostic peritoneal lavage (choice C) is not the right option for this patient, as it may not reveal pathology in the retro peritoneum (i.e., pancreas). Diagnostic peritoneal lavage is usually used in the initial investigation of the blunt abdominal trauma to rule out any intraperitoneal bleeding or evidence of bowel injury.

Continued observation (choice D) is not the right option in the presence of symptoms and signs in a patient with blunt abdominal trauma.

Ultrasonography (choice E) is not an optimum investigation method in the diagnosis of a pancreatic fracture. It is technician-dependent and subjective. Ultrasonography in blunt abdominal trauma is useful in the initial investigation to rule out any fluid collection, i.e. blood.

 

A 21-year-old man is brought by ambulance to the emergency department after being involved in a high-speed collision on his motorcycle. The patient struck a retaining wall at over 100 miles per hour. At the scene, he was non-responsive. He was intubated, a peripheral intravenous was placed, and he was transported to the hospital. On arrival, the patient is non-responsive to command. His temperature is 37.0 C (98.6 F), blood pressure is 60/30 mm Hg, pulse is 140/min, and respirations are 10/min via mechanical ventilator. He has multiple ecchymoses on his abdomen and chest, with an open femur fracture on the right and a depressed skull fracture. His abdomen is distended and tense and a radiograph suggests massive blood in the abdomen from a venous tear. A femoral vein cut-down is performed by the surgical team and a femoral vein central line is placed. After rapid infusion of 6 liters of crystalloid and 4 units of packed red cells via the femoral line, the patient is noted to be in pulseless electrical activity. The most appropriate next step in the management of this patient is to

  A. bring the patient emergently to the operating room
  B. continue rapid transfusion of blood products and crystalloid via the femoral line
  C. give epinephrine, intravenously
  D. place an upper extremity intravenous line and infuse volume and blood product
  E. perform DC cardioversion at 200 Joules
Explanation:

The correct answer is D. The utility of central lines are related to the nature of the injury or clinical condition of the patient. For example, internal jugular lines are useful when the shoulders or chest is going to be operated on. For this patient who likely has an inferior vena cava rupture, a precaval line does nothing except deliver blood product to his vena cava that promptly discharges it through a defect into the abdominal space. This patient is now in PEA because he has continued to lose volume through his defect. An upper extremity line should be placed (utilizes SVC) and volume should be delivered in that manner.

Attempting to bring the patient emergently to the operating room (choice A) could result in his death. Patients must be stabilized (the ABCs of resuscitation) before any additional interventions are taken. This patient is in an ACLS arrest and must be resuscitated prior to being discharged from the emergency department to the operating room.

For the reasons discussed above, to continue rapid transfusion of blood products and crystalloid via the femoral line (choice B) would be ineffective at restoring this patient’s pressure.

Similarly, giving epinephrine intravenously (choice C) is not correct because this is only called for in pulseless VT or VF.

DC cardioversion at 200 Joules (choice E) is incorrect because this patient is in PEA arrest. This ACLS algorithm calls for interventions that restore the circulatory tone. There is no evidence that this patient is in pulseless ventricular tachycardia or fibrillation, which does call for countershock.

 

  A 71-year-old man is undergoing a coronary artery bypass procedure in the morning. He has a long-standing history of coronary artery disease and hypertension and takes atenolol, furosemide, captopril, and digoxin daily. The patient also suffers from moderate osteoarthritis of the cervical spine. You are called to insert the internal jugular central line that will be used during the case the following morning. It is difficult to get the patient in optimal position but the attempt is made at placing the line. After multiple attempts on the right, a successful left-sided internal jugular central venous catheter is placed. The position is confirmed by chest radiograph. Three hours later, the nurse calls because the patient is now stridorous. On arrival to the room, you notice that the patient is having difficulty breathing, has audible inspiratory stridor, and has a massively swollen right neck extending to the midline and to the clavicle inferiorly. The next step in the management of this patient is to

  A. administer a bolus of furosemide, intravenously
  B. apply pressure to the right side of the neck
  C. call for a vascular surgeon emergently
  D. give nebulized beta agonist therapy immediately
  E. perform a bedside surgical opening of the neck
Explanation:

The correct answer is C. All central line placements are associated with complications. The nature and incidence of those complications are site specific. For internal jugular lines, the incidence of carotid puncture is about 5%. Most of these are trivial since the puncture occurs with the small 19G or 22G seeker needle, but in patients where the anatomy is difficult, multiple attempts at line placement often mean multiple lacerations of the carotid. This patient has bled into his neck and has nearly compressed his trachea from the pressure of the contained blood (the blood is under pressure equivalent to his mean arterial pressure). A surgeon should be called immediately for either an emergency bedside opening and decompression, or to go to the operating room emergently.

Since there is no evidence that this patient is in pulmonary edema, a bolus of intravenous furosemide (choice A) will not be helpful.

Trying to apply pressure to the right side of the neck (choice B) is incorrect and could result in the collapse of the trachea. The bleeding has already stopped secondary to the tamponade effect of the closed space, any further extrinsic compression will only further compromise his airway.

Similarly, giving nebulized beta agonist therapy (choice D) is not useful since the stridor heard is not due to bronchiolar spasm but to external compression of the extra-thoracic trachea.

Performing a bedside surgical opening of the neck (choice E) should ONLY be attempted by an experienced vascular surgeon. This is because once the tamponade is relieved, the carotid will continue to bleed and unless hemostasis can be attained, the patient runs a severe risk of bleeding to death.

 

  A 49-year-old man comes to clinic for follow up and monitoring of his oral anticoagulation levels. The patient is postoperative day 62 from a left total knee replacement. On postoperative day number 2 he suffered a pulmonary embolism. He was placed on intravenous unfractionated heparin and then oral warfarin. He was discharged home with follow-up instructions to return to the clinic for monitoring of his prothrombin time/international normalized ratio (INR) every 3 weeks. On return to the clinic today his PT/INR is found to be 22.4/7.3. His physical examination is unremarkable. The most appropriate management at this time is to

  A. admit the patient to the hospital
  B. instruct the patient to discontinue warfarin and return in 1 week
  C. instruct the patient to discontinue warfarin week until his next visit in 3 weeks
  D. give protamine sulfate, intravenously
  E. give vitamin K and follow up with the patient at his next visit
Explanation:

The correct answer is A. This patient has a supratherapeutic INR and is at great risk of spontaneous bleeding. He requires hospitalization, vitamin K administration, and if there is bleeding, fresh frozen plasma. Because he is within 2 months of a pulmonary embolism, his risk of recurrent PE off of warfarin is high and he will therefore need to be restarted on heparin as soon as his INR falls below 2.0. This management can only be accomplished in the hospital.

Although withholding warfarin is appropriate, not admitting the patient to the hospital (choice B) and (choice C) is incorrect for the reasons stated above.

Protamine sulfate (choice D) is an antagonist of unfractionated heparin. It has no effect on warfarin, which interferes with the carboxylation of factors 2, 7, 9, and 10.

Again, administration of vitamin K (choice E) is appropriate in this setting, but because the patient’s risk of bleeding is so high, he should be admitted to the hospital for the management.

 

A 57-year-old woman with diabetes and hypertension comes to the office because of a 5-day history of weakness and lethargy. The symptoms developed slowly but have worsened over the last 2 days. She denies any chest pain, dyspnea, fever, cough, abdominal pain, or dysuria, but has noticed a decrease in her urine output. One week prior to admission she underwent a CT scan of the abdomen with intravenous contrast for routine follow up of an abdominal aortic aneurysm, which is stable at 4 cm. She regularly takes hydrochlorothiazide, glyburide, captopril, and aspirin. Her temperature is 37.0 C (98.6 F), blood pressure is 165/94 mm Hg, pulse is 92/min, and respirations are 14/min. Physical examination shows 2+ pitting edema in the lower extremities and 1+ peripheral pulses. Urinalysis shows trace protein, 1-3 white blood cells, tubular casts and no red casts. Laboratory studies show:

This condition most likely would have been prevented by

  A. administering acetylcysteine prior to the contrast for the CT scan
  B. holding her captopril 2 days prior to the CT scan
  C. giving her intravenous ampicillin during the CT scan
  D. giving her intravenous furosemide 1 hour prior to the CT scan
  E. premedicating her with nifedipine 2 days prior to the CT scan
Explanation:

The correct answer is A. The patient is suffering from acute tubular necrosis (ATN) presumably secondary to the contrast she received during her CT scan. Contrast induced ATN is more common in patients with diabetes and baseline renal insufficiency. The urinalysis is also suggestive of this diagnosis with the presence of tubular casts and the absence of red cell casts. Recent studies have shown that the incidence of contrast nephropathy can be reduced with the use of acetylcysteine prior to the contrast.

There is no evidence to suggest that holding captopril (choice B) will reduce the incidence of contrast induced nephropathy. ACE inhibitors have been shown to slow the progression of renal disease in diabetics with proteinuria.

Premedication with antibiotics (choice C) has no role in the prevention of contrast nephropathy. They are indicated in the prophylaxis of bacterial endocarditis in patients with valvular heart disease undergoing selected invasive procedures.

Diuretics (choice D) have not been shown to be beneficial in the prevention of contrast nephropathy. Adequate hydration should be provided to patients at risk prior to studies with contrast. Of course, if patients develop clinical evidence of volume overload diuretics should be used judiciously.

Calcium blockers (choice E) have also not shown any benefit in the prevention of ATN and should not be used for that purpose. If the patient is already taking them however, there is no reason to discontinue them prior to a contrast study.

 

A 44-year-old man with metastatic liver cancer requires a central line for total parenteral nutrition. The patient was otherwise healthy until 3 months ago at which time he was diagnosed with liver cancer. A subsequent workup for metastatic disease disclosed that the tumor had already spread to his lungs, abdominal viscera, and brain. He is scheduled to begin chemotherapy and radiation therapy and will require nutritional support. The patient is given informed consent and the details of the procedure are discussed with him. A decision is made for a right subclavian line. The patient is positioned, prepped, and draped in a sterile manner and the skin is anesthetized with 1% lidocaine. During the procedure, the guidewire slips from your fingers and disappears through the lumen of the catheter. This patient is at greatest risk for

  A. atrial-septal perforation
  B. cardiac arrhythmia
  C. pneumothorax
  D. tricuspid valve damage
  E. ventricular perforation
Explanation:

The correct answer is B. Wire loss during insertion of a central venous line is a relatively common occurrence with an estimated incidence of about 1%. Once lost, the issue becomes where the wire will become lodged. The most common location is in the distal pulmonary artery such that the proximal tip of the wire is in the right ventricle. When this occurs, it is not uncommon, in fact it is quite normal to see arrhythmia. The problem is when these rhythms are ventricular tachycardias. The wire must be removed by an interventional radiologist or cardiac surgeon.

Unless the wire lodges in the atrium and it is somehow “pushed” during extraction, atrial-septal perforations (choice A) are uncommon.

A pneumothorax (choice C) is not a complication of wire loss, but of placing subclavian central lines.

Again, unless some force is applied to the wire during attempted extraction, tissue damage to the tricuspid valve (choice D) is not very common.

A ventricular perforation (choice E) could occur if the wire is coiled in the ventricle. The force of ventricular contraction could force the tip of the wire through the membranous septum. Although this is possible, the most common, in fact, routine complication from wire loss is cardiac arrhythmia.

 

An 11-year-old boy is rushed into the emergency department following a motor vehicle accident. The patient was a restrained, front seat passenger when an oncoming car hit the passenger side of the car. The boy denies loss of consciousness, but in the hospital he complains of pain over his right side. His pulse is 139/min, blood pressure is 118/59 mm Hg, and respirations are 24/min. On physical examination he has decreased breath sounds over the right base and there is ecchymosis over the right flank. His abdomen is soft with tenderness in the right upper quadrant. Appropriate management of his airway with neck stabilization is provided and he is resuscitated appropriately. Imaging studies of his neck are negative and a chest X-ray does not show a pneumothorax or rib fracture. Dipstick of spontaneously voided urine is positive for blood. Urinalysis confirms the presence of 50 RBCs/hpf. In regards to his hematuria the next most appropriate course of action is to

  A. begin empiric antibiotics
  B. obtain CT scan of abdomen and pelvis
  C. order renal/bladder ultrasound
  D. perform retrograde urethrogram
  E. place Foley catheter
  F. repeat urinalysis
  G. schedule outpatient intravenous pyelogram (IVP)
Explanation:

The correct answer is B. The management of hematuria associated with trauma differs in adults and children. In the adult population, imaging is performed only in those patients with gross hematuria or microscopic hematuria plus hypotension. This differs from the pediatric patient. In children, any degree of hematuria (gross or microscopic) should be investigated with imaging studies. One reason for this discrepancy is that large amounts of catecholamines released in injured children may sustain blood pressure in the face of hypovolemia. A CT scan is the most useful imaging modality in this setting. A CT is noninvasive, accurate and fast, and it can help in assessing the size and extent of retroperitoneal hematomas and renal parenchymal trauma. Not only does this child have microscopic hematuria (an indication by itself to perform imaging studies), but he also has signs (flank ecchymosis and tenderness) that raise the suspicion of renal injury. High suspicion for renal injury (i.e., rib fracture, flank contusion, deceleration injury) is another indication to perform imaging studies.

There is no indication at this time that the patient requires antibiotics (choice A). He has no open fractures, large abrasions, or burns. Further diagnostic studies must be undertaken to determine if the patient requires antibiotics for any disruption to the urinary system.

Ordering a renal/bladder ultrasound (choice C) acknowledges the fact that the patient does require investigation of his urinary system. However, a CT scan is quicker and will have a much higher yield for associated urinary and abdominal injuries versus ultrasound.

A retrograde urethrogram (choice D) allows for visualization of the urethra to investigate for extravasation. It is performed when there is gross blood at the meatus. It will not help in this patient whose injuries are suspected to be intraabdominal.

Hematuria is not an indication for Foley catheter placement (choice E). As long as the patient is awake without altered sensorium he should be given an opportunity to void on his own (which he has done). It should be noted that in any patient who has gross blood at the meatus a retrograde urethrogram must be obtained prior to placing a Foley catheter.

Repeating the urinalysis (choice F) will only delay this patient’s work-up. Although the urine dipstick may be falsely positive for heme, there is no reason to doubt the validity of the urinalysis. Besides, this patient’s flank ecchymosis and tenderness warrant imaging independently of his urinalysis results.

Intravenous pyelogram, or IVP (choice G), has a role in evaluating the urinary tract. However, with the use of CT scan, IVP has a very limited role in evaluating renal/ureteral trauma. Delaying imaging until this patient is an outpatient could mean missing a potentially life-threatening renal injury.

 

  A 29-year-old truck driver is transferred to the general medical floor from the intensive care unit where he was being treated after sustaining multiple fractures from a motor vehicle accident. It is determined that the patient was under the influence of alcohol at the time of the accident. He has been in the intensive care unit for 20 days following open reduction and internal fixation of bilateral femur, tibia, and fibula fractures. The patient is otherwise doing well and has full use of his upper body. Legal proceedings will begin subsequent to discharge, and the patient states he is committed to ceasing alcohol consumption. Past medical history is significant for alcoholism. Medications include imipenem and morphine administered via patient-controlled analgesic pump. At this time, the patient is at greatest risk for

  A. alcohol withdrawal
  B. deep venous thrombosis
  C. fat emboli to lungs
  D. narcotic addiction from the morphine
  E. suicide
Explanation:

The correct answer is B. This patient is profoundly immobile due to multiple fractures of both legs and he is at increased risk for developing a thrombus in the veins of his lower extremities. Deep venous thrombosis (DVT) prophylaxis with subcutaneous heparin should already have been started. DVT prophylaxis is necessary in all immobile patients, especially those who have sustained major orthopedic injury.

Alcohol withdrawal (choice A) usually occurs a few hours to 10 days after the last drink. Most typically, it occurs in the first 48 hours. Symptoms include acute psychosis with hallucinations, delusions, disorientation, agitation, and tremors. In this patient, alcohol withdrawal is very unlikely to start now, 20 days after the injury.

A fat emboli (choice C) from bone fractures occurs in close temporal proximity to the fracture or to the surgery. The patient is now 10 days postoperative and 20 days after the initial injuries, making this unlikely.

Narcotic addiction (choice D) from patient controlled analgesia is very rare and allows the patient to participate in his medical care. Care must always be taken to balance analgesia and addiction potential.

Suicide (choice E) is unlikely given the hopeful outlook of this patient and lack of history of depression or suicidal tendencies.

 

A 70-year-old man with hypertension, diabetes mellitus, and hypercholesterolemia is postoperative hour number 8 after a laparoscopic cholecystectomy. He was diagnosed with cholecystitis 2 days earlier and underwent an uneventful procedure. Now the patient is on the floor for postoperative observation. His medical history is remarkable for severe ischemic cardiomyopathy and he is status-post 2 myocardial infarctions. He has known congestive heart failure with an ejection fraction of 70%. He also has severe aortic stenosis with a valve area of less than 0.5 cm2. He takes maximal doses of a beta-blocker and calcium channel blocker daily, as well as enalapril and furosemide. Preoperatively his temperature was 38.0 C (99.6 F), blood pressure was 180/85 mm Hg, pulse was 44/min, and respirations were 18/min. Intraoperatively his pulse was stable at 40/min and his blood pressure was 150/70 throughout the case. On arrival to the floor, you give him 0.1mg of atropine sulfate intravenously to treat the bradycardia. His pulse rises to 140/min with a blood pressure of 120/40 mm Hg. He begins to complain of severe shortness of breath and becomes mildly hypoxic on room air. The most appropriate therapy at this time is to

  A. administer edrophonium, intravenously
  B. administer furosemide, intravenously
  C. administer propranolol, intravenously
  D. obtain a chest radiograph and a 12 lead electrocardiogram
  E. perform endotracheal intubation
Explanation:

The correct answer is C. This patient has severe diastolic dysfunction secondary to his hypertension, severe aortic stenosis, and ischemic heart disease. This can be appreciated by the fact that he has known congestive heart failure and has an ejection fraction of 70%, greater than normal for his age. This represents a smaller end-diastolic volume with preserved stroke volume. These patients are prone to dramatic increases in pulmonary pressures with tachycardia of any cause. They are therefore prone to pulmonary edema and congestive failure episodes with tachycardia. This is the reason why he is so heavily beta blocked. There is no indication to treat a sinus bradycardia that is without symptoms. The patient must now have his heart rate slowed with beta-blockers (propranolol).

To antagonize the atropine, intravenous edrophonium (choice A) should not be considered. Such a combination is used for reversal of neuromuscular blockade during anesthesia, but can have severe effects such as a dramatic bradycardia and even asystole in patients who are heavily beta blocked and on nodal agents such as calcium channel blockers, as in this patient.

Intravenous furosemide (choice B) is appropriate management for the pulmonary edema once the precipitating cause of the edema is reversed. Diuretics lower venous filling pressures and therefore decrease pulmonary congestion.

Obtaining a chest radiograph and a 12 lead electrocardiogram (choice D) does not take precedence over the acute management of the patient’s symptoms. If there was uncertainty as to the cause of the symptoms, these tests may be able to clarify the cause and therefore direct therapy.

Although this patient is in mild respiratory distress from the pulmonary edema, there is no evidence of severe hypoxia, hypercarbia, obtundation, or altered mental status that dictates the need to perform tracheal intubation (choice E).

 

A 64-year-old farmer comes to the clinic with an injury to the foot, which happened at his farm 24 hours ago. He reports that he was working on the farm, when he accidentally stepped on a rusty nail, which penetrated deep into his foot. He took some analgesics and he was feeling well. His wife urged him to come to the clinic. He denies any fever, chills, or rigor. His past medical history is significant for non-insulin dependent diabetes mellitus, which is well controlled with oral therapy. He has no past surgical history. Examination of the foot reveals a deep penetrating wound in the sole of the left foot. There is no associated erythema or induration around the foot. No foreign particles are noted along the edges of the foot, which is tender on palpation. There is no motor or sensory loss. No active bleeding is noted in the penetrating wound. The patient tells you that he had 3 doses of tetanus toxoid injections when he was young. The last tetanus injection was 8 years ago for a similar episode. The most appropriate next step in management is to

  A. administer a tetanus toxoid booster injection
  B. administer a tetanus toxoid booster injection and human immunoglobulin
  C. provide human immunoglobulin
  D. provide no additional therapy at this time
  E. surgically debride the wound
Explanation:

The correct answer is A. Tetanus prone wounds are any wounds that are over 6 hours old, deep, crushed or penetrated, contaminated with soil, associated with compound fractures, partial or full thickness burns, or human or animal bites. Patients who received 3 doses of tetanus toxoid in the last 5 years do not need further therapy for a tetanus prone wound or for a clean wound. Patients who received a tetanus toxoid between 5-10 years ago and have a tetanus prone wound need booster tetanus toxoid injections, whereas those with a clean wound would need no further therapy. When the last dose of tetanus toxoid was more than 10 years ago, then both clean wounds and tetanus prone wounds need tetanus toxoid booster injections. In addition, those with a tetanus prone wound, whose last dose was more than 10 years ago, need to have human immunoglobulin (choice C) administered.

When the tetanus toxoid vaccination history is not available or the tetanus toxoid injections were administered more than 10 years ago, then an individual with a tetanus prone wound would need tetanus toxoid and human immunoglobulin (choice B). Whereas in a clean wound, a tetanus toxoid complete course is essential. In this case, the farmer received a tetanus toxoid booster 8 years ago and the wound described is a tetanus prone wound. Hence, one booster dose of tetanus toxoid is indicated in this patient.

No therapy (choice D) is not the correct option in this patient, as the wound is tetanus prone, and the last toxoid booster was given more than 5 years ago.

Surgical debridement is indicated in crushed wounds. Wherever there is a soft tissue injury from crushed wounds, and when there is some devitalized tissue, surgical debridement is indicated, even if it is old. In a clean, penetrating wound, surgical debridement (choice E) is not indicated.

 

A 19-year-old boy comes to the office because of a cough for the past 2 weeks. It was initially a “dry, annoying cough,” but it has recently changed to a “wet cough with yellowish-green sputum.” He is a college freshman and is “pledging” a fraternity. He admits to many nights of “hard drinking.” You notice that his front tooth is missing and he says that he lost it 2 weeks ago during a “hazing” event when the pledges were forced to “funnel” beer. Her temperature is 37.8 C (100.0 F), blood pressure is 110/80 mm Hg, and respirations are 20/min. Physical examination shows wheezes over the right lung base. It is otherwise unremarkable. The most appropriate next step is to

  A. admit him to the hospital for immediate bronchoscopy
  B. order a chest x-ray
  C. order pulmonary function tests
  D. order tomography of the right lower lobe
  E. prescribe antibiotics and send him home
Explanation:

The correct answer is B. This patient most likely has a foreign body aspiration (his tooth). A foreign body aspiration typically presents with a new cough, wheezing and the patient often has a history of heavy drinking, drug overdose, or trauma. This patient was “funneling” beer, which is when a large funnel and a tube are used to supply a large amount of beer into the recipient’s mouth at a rapid speed. A chest x-ray is the initial study and may show a radiopaque object and postobstructive pneumonia or abscess. Management is by bronchoscopy combined with antibiotics.

Admitting him to the hospital for immediate bronchoscopy (choice A) is inappropriate because while it seems likely that he has a foreign body aspiration (his tooth), a chest x-ray should be ordered first. He should be evaluated for a radiopaque object and postobstructive pneumonia, abscess or another process that may be causing these symptoms.

Pulmonary function tests (choice C) are inappropriate at this time. This patient most likely has a foreign body aspiration, not asthma. The wheezing that is heard over the right lower lobe is most likely caused by obstruction of a bronchus.

Tomography of the right lower lobe (choice D) is not the next step in evaluating this patient who most likely has a foreign body aspiration. A chest x-ray should be ordered.

Since this patient most likely has a foreign body aspiration, it is inappropriate to prescribe antibiotics and send him home (choice E). He requires removal of the object and antibiotics. Sending him home with antibiotics may be appropriate if he is a healthy patient with a cough and a suspected mycoplasma pneumonia infection, however a chest x-ray is usually indicated in this situation too.

 

A 45-year-old man is struck by a motor vehicle and is transported by a Med-Flight to the local emergency department. The patient is reported to be intoxicated with a Glasgow score of 15 on the scene of the accident. He was struck by a vehicle while crossing the street. On arrival, the patient is awake and somewhat combative. He is alert to person only. Primary survey reveals a well-developed man in mild distress. He is in a cervical collar. His blood pressure is 150/90 mm Hg, his heart rate is 130 /min, and he is breathing at 26/min. He has obvious lower extremity tibia fractures bilaterally and a laceration on his forehead. Secondary survey reveals severe pelvic trauma with bilateral inferior and superior pubic ramus fractures. The most appropriate diagnostic test at this time is

  A. abdominal CT scan
  B. abdominal ultrasound
  C. diagnostic peritoneal lavage
  D. exploratory laparotomy
  E. pelvic ultrasound
Explanation:

The correct answer is A. This patient has severe trauma involving his lower extremities and pelvis. The concern is for intraabdominal trauma resulting in bleeding. An abdominal CT scan is the imaging modality of choice since it detects both intraabdominal and retroperitoneal blood.

An abdominal ultrasound (choice B) has been shown to be equivalent to diagnostic peritoneal lavage (choice C) for detection of free intraabdominal blood. However, neither of these modalities can reliably diagnose the presence of a retroperitoneal bleed.

Although some centers immediately opt for an exploratory laparotomy (choice D), this is actually not the appropriate diagnostic step. This patient will certainly go to the operating room for repair of his tibial fractures and stabilization of his pelvis. The addition of a midline abdominal incision to his procedure list would entail a tremendous amount of OR time and additional chance of morbidity for the patient. The most appropriate intervention is to look for the presence of blood and if the clinical picture still suggests a bleed, but there has been none disclosed by imaging, then a surgical exploration is warranted.

A pelvic ultrasound (choice E) offers no diagnostic benefit for the evaluation of an intraabdominal bleed.

 

A 3-year-old recently adopted boy is brought to the office for the first time by his new parents for a “check-up.” They say that he seems to be adjusting to his new family “pretty well”, but that he is a bit “slower” than their other children and he has poor socialization skills. He is able to walk unassisted. However, he cannot walk up steps (even though his previous home had steps), and he is only able to say a few words. They are also concerned because he looks a little different from their other kids, who have started to make mean comments to him. Physical examination shows microcephaly, epicanthal eye folds, small teeth, poorly formed concha, and a holosystolic murmur. This condition could most likely been prevented by

  A. dietary supplementation with vitamin D when he was an infant
  B. maternal avoidance of alcohol during pregnancy
  C. maternal avoidance of cocaine during pregnancy
  D. maternal intake of folate before and during pregnancy
  E. regular consumption of iron-containing formula when he was an infant
Explanation:

The correct answer is B. This patient most likely has fetal alcohol syndrome (FAS), which could have been prevented by maternal avoidance of alcohol during pregnancy. The classical features of this syndrome are growth retardation, abnormal facial features, and central nervous system problems. Patients affected with FAS can have serious lifelong disabilities, including mental retardation, behavioral problems, and learning disabilities. It is unknown exactly how alcohol causes these specific abnormalities. However, it is important to stress to all patients that no amount of alcohol should be consumed during pregnancy. A nurturing and stable environment is especially important for children with FAS.

Dietary supplementation with vitamin D when he was an infant (choice A) is incorrect because vitamin D deficiency is associated with rickets, not FAS. Rickets is characterized by a delayed closure of the fontanelles, craniotabes, bowing of the lower extremities, and fractures.

Maternal avoidance of cocaine during pregnancy (choice C) is incorrect because maternal cocaine usage is associated with hyperirritability, cerebral infarction, and intrauterine growth retardation. It is not associated with the craniofacial abnormalities in this case.

Maternal intake of folate before and during pregnancy (choice D) is important to reduce the risk of neural tube defects (NTD). It is recommended that women of reproductive age take folate daily. The patient in this case does not have a NTD.

Regular consumption of iron-containing formula when he was an infant (choice E) would have prevented iron-deficiency anemia, which is associated with fatigue, anorexia, irritability, pale skin and mucous membranes, and a decrease in cognitive function. Mouth soreness and “spooning” of the nails may be present with severe anemia. Iron-deficiency anemia is not associated with abnormal facial features.

 

A 30-year-old man who you have not seen in a few years comes to the clinic because of a recent psoriasis flare up. You are talking about possible new stressors in his life that might have precipitated this worsening. The patient agrees and tells you that he has been traveling a lot recently. He explains that an “odd” thing happened when he went to visit his ex-girlfriend who lives in another state. He ended up being admitted to the psychiatric hospital because he was acting as if he were on “speed”. He was diagnosed with bipolar disorder and treated for several days. He was instructed to follow up with a mental health center in his community and given an appointment for the next week. Currently, he feels upset because he was trying to get back to his routine and feeling confident. As his doctor, you are aware that you will need to encourage his compliance, because the worsening of psoriasis could be ascribed to the medication he was placed on. The medication that is most likely causing his skin condition is

  A. carbamazepine
  B. haloperidol
  C. lithium carbonate
  D. olanzapine
  E. valproic acid
Explanation:

The correct answer is C. Side effects that occur with lithium carbonate are often troublesome. These include sedation, cognitive difficulties, dry mouth, hand tremor, increased appetite, polydipsia, polyuria, nausea, diarrhea, psoriasis, and acne.

Carbamazepine (choice A) can cause aplastic anemia and agranulocytosis as the most serious side effects. The most common side effects include dizziness, ataxia, sedation, dysarthria, nausea, hyponatremia, and cardiovascular conduction problems. Rarely can it cause rash and exfoliation.

Haloperidol (choice B) related side effects generally are a consequence of the blockade of dopaminergic D2 receptors. Apart from a spectrum of extrapyramidal symptoms, side effects include sedation, weight gain, hyperprolactinemia, and neuroleptic malignant syndrome.

Olanzapine (choice D) is a typical antipsychotic indicated in the management of psychotic disorders and mood disorders because of its mood stabilizing properties. It causes significant sedation, weight gain, postural hypotension, dizziness, dry mouth, constipation, and other anticholinergic side effects. It rarely causes vesiculobullous rash.

Valproic acid (choice E) most commonly causes short-term nausea, vomiting, diarrhea, sedation, dizziness, and tremor. Alopecia and weight gain are long-term side effects.

 

  A 9-year-old girl is brought to the office by her mother because it always seems as if she is going to “burst from all of her energy.” Her teacher says that she has difficulty following instructions and waiting to be “called on” when a question is posed to the entire class. This child has been disrupting the class for “months” with her impulsive speech and “constant motion.” “The final straw” was when she knocked over 3 buckets filled with colored sparkles when she was supposed to be sitting quietly at her desk. The mother is concerned that she is going to be seriously injured “one of these days” because of her distractibility and carelessness. It is almost “impossible” to get her to go to bed at night. As the mother goes over her history, the child is going through the drawers in the examining room. You explain that she probably has attention-deficit hyperactivity disorder and you prescribe a high dose of methylphenidate. You educate the patient and her mother about the most common side effects of the medication, and tell them to call you if any of them occur. Two weeks later, you receive a message that the mother called the office because she believes that her daughter is experiencing a “very frustrating side effect” of the medication. The most likely side effect that she is referring to is

  A. constipation
  B. dystonia
  C. insomnia
  D. sedation
  E. weight gain
Explanation:

The correct answer is C. Methylphenidate is one of the most common drugs prescribed for attention-deficit hyperactivity disorder (ADHD). It is a psychostimulant that produces positive behavioral responses in the majority of individuals. Common side effects during the initial therapy include insomnia, loss of appetite, abdominal pain, and weight loss. Growth suppression with weight gain and/or inhibited body growth has been reported with the long-term use of methylphenidate in children.

Constipation (choice A) is a common side effect of high potency antipsychotic agents and other drugs with anticholinergic effects. It is not typically seen with stimulants, such as methylphenidate.

Dystonia (choice B) is a side effect of high potency antipsychotic agents. It is not a common side effect of methylphenidate.

Insomnia, not sedation (choice D), is a common side effect of methylphenidate.

Weight loss is typically reported during treatment with methylphenidate, not weight gain (choice E).

 

A 71-year-old man is postoperative day number 12 from a heart transplant. The patient has a long-standing history of ischemic cardiomyopathy and successfully underwent a 5-hour transplant from a 22-year-old donor. During the procedure he was started on his immunosuppressive therapy that has continued. In the immediate postoperative period he did well. He was extubated on day number 1, had his pulmonary artery and radial arterial catheters removed on day number 3, and was transferred from the coronary care unit on day number 4 with excellent pain control. Over the past 3 days however, he has had increasing fever, lethargy, and mediastinal tenderness. The sternal wound appears mildly erythematous but nonsuppurative. On palpation, the incision site is tender. The most appropriate therapy is at this time is

  A. broad-spectrum antibiotics
  B. decreasing dose of immunosuppressive drugs
  C. initiate antifungal therapy
  D. surgical debridement
  E. there is no therapy indicated
Explanation:

The correct answer is A. This patient likely has mediastinitis, a common postoperative complication after heart transplant. The combination of the nature of the wound (median sternotomy) with the profound immunosuppression creates a furtive environment for mediastinal infection. Such patients require broad-spectrum antibiotic therapy.

Because of the transplant and the possibility of acute-rejection, decreasing the dose of immunosuppressive drugs (choice B) is not tenable.

Bacteria cause most mediastinal infections in the immediate and intermediate postoperative periods. Therefore, fungal infections in transplant recipients and the need for antifungal therapy (choice C) are not usually an issue until 6-9 months of immunosuppressive therapy.

Surgical debridement (choice D) is usually required when the sternum undergoes dehiscence or there is necrotic tissue present that can become a culture medium for additional infections. There is no indication that this patient’s infection is that severe.

Because mediastinitis is a serious infection and can cause sternal dehiscence and chronic infection, antibiotic therapy is indicated despite concerns for selecting for resistant organisms (choice E).

 

  You are evaluating a 34-year-old man in the emergency department following an approximately 8 foot fall from a scaffolding while at work . He lost consciousness at the scene, but regained consciousness and was hemodynamically stable on transfer, as well as, upon arrival in the hospital. On arrival, he was awake, but complaining of left-sided chest pain and left-sided facial pain. Of note, the patient has a history of alcohol and substance abuse, and reports that he drank 3 beers this morning prior to work. His physical examination is remarkable for normal mental status, some left zygomatic abrasions, as well as abrasions below the left eye. Head, eyes, ears, nose, and throat examination is otherwise normal. Chest examination shows some left rib tenderness. Laboratory findings include an alcohol level of 365 and a serum positive for cocaine. Complete blood count, electrolytes, and coagulation studies are normal. X-rays show no fractures. The most appropriate next step in management is to

  A. discharge him to home from the emergency department
  B. discharge him to a substance abuse rehabilitation center from the emergency department
  C. get a CT scan of the head
  D. get an MRI of the spine
  E. immediately start diazepam for alcohol withdrawal
Explanation:

The correct answer is C. This patient has had head trauma with loss of consciousness. Although his neurologic examination is normal, a head CT should still be done to look for a bleed. These are often subclinical on admission and worsen over the first 1-2 days.

This patient sustained a head injury with loss of consciousness and requires evaluation to determine if there is an intracranial bleed. He should not be discharged home from the emergency department (choice A) without a more thorough evaluation.

While substance abuse rehabilitation (choice B) may be necessary in the future, right now he needs to be evaluated for an intracranial bleed and so he should not be discharged at this time.

There is nothing in the history or physical examination to suggest a spinal abnormality, so a spine MRI (choice D) is not indicated.

He should be given a benzodiazepine for alcohol withdrawal (choice E), however this is not urgent and can wait until the acute issues have been addressed.

 

A 39-year-old man is two days postoperative from a thyroid resection for papillary carcinoma of the thyroid. Prior to his diagnosis, his medical history was unremarkable, but 1 year prior to his surgery, he presented to his primary physician for evaluation of a lump in his neck. This lump was eventually diagnosed as a cancerous thyroid nodule. The patient was admitted to the hospital and his surgery went uneventfully. On visiting him postoperatively, both you and the patient notice that his voice is hoarse. The most likely reason for this is

  A. damage to the recurrent laryngeal nerve
  B. damage to the vocal cords
  C. new tumor growth
  D. residual tumor
  E. upper respiratory infection
Explanation:

The correct answer is A. Knowledge of anatomy is critical in many areas of patient care. This is especially true in surgery and in managing postsurgical complications. This patient had work on the thyroid. The recurrent laryngeal nerve innervates all but one of the muscles of the larynx and this nerve is adjacent to, if not directly opposed to the thyroid gland. It is often damaged during thyroid resections and in fact, disclosure of such possible damage is standard during the surgical consent process.

Although damage to the vocal cords (choice B) secondary to the endotracheal tube could also account for the symptoms, such occurrences are rare and when they do occur, happen as a result of prolonged intubation with an inappropriately large endotracheal tube.

In 48 hours it is unlikely that there is new tumor growth (choice C).

Unless the original tumor was causing vocal cord paralysis by impinging upon the recurrent laryngeal nerve, any residual tumor (choice D) should not either.

This patient has no evidence of any upper respiratory infection (choice E). Even if he did however, given his proximity to surgery of the thyroid, the much more likely explanation is that his hoarseness is secondary to his surgery or anesthesia, not a URI.

 

A 64-year-old woman with a diagnosis of chronic paranoid schizophrenia has moved recently to town to live with her sister. She comes to the clinic for the first time, since she needs to have a new doctor and needs her medication. She has been healthy most of her life, and except for left hip replacement surgery, has had no other interventions or treatments. She has been maintained well for years on thioridazine. She has not been in the hospital for the past 15 years. She hears voices occasionally, but has developed strategies to distract herself. She shares that she has not really had any other problems. She goes to church, walks her sister’s dog and watches TV. She denies any current complaints. Her physical examination is unremarkable. Given this patient’s history of treatment and current age, the effect of thioridazine that she should be evaluated for is

  A. cataracts
  B. diarrhea
  C. hypersalivation
  D. hypertension
  E. prolongation of the Q-T interval
Explanation:

The correct answer is E. Thioridazine belongs to the phenothiazine group of antipsychotics that can cause prolongation of Q-T interval, inversion of the T wave, and sometimes a bifid T or U wave. Changes are reversible. It has been noted that several sudden deaths occurred secondary to cardiac arrests in patients treated with thioridazine. The use of EKG periodically is thus advised.

Cataracts (choice A) have been described in beagle dogs treated with quetiapine. Thioridazine can cause pigmentary retinopathy that is dose dependent. Because of its anticholinergic effects, it may precipitate a glaucoma attack.

Diarrhea (choice B) is not a usual side effect of thioridazine. On the contrary, it has anticholinergic properties, thus causing dry mouth, constipation, and paralytic ileus.

Hypersalivation (choice C) is a side effect of clozapine. Thioridazine, because of its anticholinergic effects, causes dryness of the mouth and other mucous membranes.

Hypertension (choice D) is not a side effect of thioridazine. All phenothiazines tend to cause hypotension, secondary to the action on alpha-adrenergic receptors.

 

  A 74-year-old woman with diabetes and osteoarthritis is seen in your office for a postoperative medical visit. She has been a long-time patient of yours and five days ago underwent a left total knee replacement. Her hospital course was uneventful and she comes to the office today for a follow-up on her medical regimen. She continues to take her daily NPH insulin with good control of her blood sugars. She also continues to take her oxycodone for pain that was given to her in the hospital. She is involved in a physical therapy rehabilitation program at the local hospital. The medication that is most indicated for this patient at this time is

  A. aspirin
  B. enalapril
  C. ibuprofen
  D. subcutaneous unfractionated heparin
  E. warfarin
Explanation:

The correct answer is E. This patient is post a total knee replacement and is currently not on any anticoagulation therapy. The risk of deep venous thrombosis and subsequent pulmonary embolism is very high in this population and it is the standard of care to initiate warfarin or low molecular weight heparin postoperatively for a period of 6 weeks to 6 months.

Oral aspirin (choice A) is an anti-platelet agent that has no role in the prevention of DVT.

An ACE inhibitor (choice B) is, in the long-term, an excellent drug for this patient given her diabetes. However, in the post-surgical period, the drug that needs most consideration is for the most pressing issue.

A non-steroidal antiinflammatory agent (choice C) does not appear to be indicated at this time as the patient appears to have reasonable pain control with her opiate.

Subcutaneous unfractionated heparin (choice D) is used for the prevention of DVT in immobile or hospitalized patients unable to ambulate. However, the efficacy of post-orthopedic surgery, especially after joint procedures, is very poor given the increased venous stasis.

 

  A 61-year-old woman is status post a right total hip replacement 3 hours ago. She underwent an uneventful replacement with hardware under spinal anesthesia. She is brought the postanesthesia care unit (PACU) sedated but alert and oriented to person, place, and time. Her past medical history is significant only for hypertension and gout for which she takes allopurinol and atenolol daily. On arrival to the PACU, she complains of some mild shortness of breath and chest pain. Over the past 3 hours, her shortness of breath significantly worsens and she has pleuritic chest pain on her right side. Her temperature is 37.0 C (98.6 F), blood pressure is 100/60 mm Hg, pulse is 128/min, and respirations are 32/min. She appears markedly dyspneic, but is alert and oriented to person, place, and time. Physical examination is remarkable for clear lung fields and jugular venous pulse visible at 12cm with the patient at 30 degrees elevation. There is no chest wall tenderness on palpation. The most appropriate immediate action is to

  A. administer a propranolol, intravenously
  B. administer morphine for pain control
  C. give the patient supplemental oxygen by face mask
  D. order a chest radiograph
  E. start warfarin therapy
Explanation:

The correct answer is C. If a transesophageal echo probe is placed in every patient undergoing hip-fracture repair, the incidence of fat and particle debris in the right atrium approaches 70%. In fact, a major risk of lower extremity orthopedic procedures is pulmonary embolism due to fat or clot. The intramedullary pressures generated during the repair are greater than 500psi and are enough to cause venous extrusion of fat and other particulate matter into the circulation. This patient almost certainly suffered a fat embolism and her hypotension, elevated neck veins, tachycardia, and dyspnea reflect this fact. She requires supplemental oxygen and possibly endotracheal intubation since this syndrome will slowly progress over the ensuing 24 hours.

Giving the patient propranolol (choice A) for her tachycardia could be fatal . Her tachycardia is a response to the low filling of the left ventricle secondary to fat embolism present in her pulmonary arterioles and venules. Her blood pressure is already tenuous despite her tachycardia and without it her cardiac output and therefore blood pressure would likely drop precipitously. There is no evidence that this patient has rate-related cardiac ischemia and therefore is hypotensive.

Giving the patient morphine for pain control (choice B) is not appropriate. The patient is not complaining of surgical site pain, but rather she is manifesting pain from a pleuritic component of her embolism. Treating this pain with morphine before the patient is stabilized would result in respiratory collapse since the morphine would depress her respiratory rate and therefore her oxygenation.

Ordering a chest radiograph (choice D) is appropriate to evaluate for any pulmonary edema or obvious pulmonary infarction only after the patient has received oxygen. Patients should be treated and stabilized prior to diagnostic interventions.

Warfarin therapy (choice E) is incorrect in this case because this patient did not suffer a pulmonary embolism due to clot, but from fat. Clot emboli are an important source of morbidity and mortality in this population, but usually beginning 24 hours post-procedure. This patient has had an acute perioperative event that is too soon to be clot.

 

  A 27-year-old man comes to the emergency department 30 minutes after stepping on a nail in his garage. He was running late for his tennis game and he was about to get into his car when he felt a sharp, dagger-like feeling in his left foot. He looked down and saw a 3 inch nail protruding through the sole of his tennis sneaker. He ran back into the house and had his wife drive him to the hospital. He pulled the nail out of his foot in the car and says that the nail appears intact. He says that he is generally very healthy and has not been to the doctor in “ages”. He cannot even remember his last “check-up”, but he assumes that it was when he was in his “late teens.” He does not know his immunization history but he recalls that he had all of the recommended vaccines before going to college. Physical examination shows a clean, stellate puncture wound on his left heel that appears to only penetrate the superficial epidermis. There is no swelling or pain with the movement of the toes and sensation is intact. He shows you the intact, shiny, clean nail. After you irrigate, debride, and carefully inspect the wound, the most appropriate next step is to

  A. advise him to soak his foot in an iodine solution at home
  B. give him a tetanus and diphtheria toxoid
  C. order a CT scan of his left foot
  D. request an immediate orthopedic surgery consult
  E. send him home with a prescription for an antibiotic that covers Pseudomonas aeruginosa
Explanation:

The correct answer is B. This patient has a puncture wound through a sneaker, which is a very common presentation in the hospital and on the boards. The major considerations in this case are immunization history and penetration through the sneaker (risk of Pseudomonas aeruginosa infection). Since he has not been to the doctor in a while, he probably needs a tetanus and diphtheria toxoid at this time. This is recommended over just the single antigen tetanus toxoid for persons >7 years old. The tetanus immune globulin is an antitoxin that is usually only recommended for contaminated, severe wounds, and when the patient is unsure if they have ever received a tetanus vaccination or have an incomplete immunization history. It should be given in this case because it is also recommended for puncture wounds. Since this patient was up-to-date on his immunizations before he went to college, approximately 10 years ago, he requires the Td toxoid. The issue of a Pseudomonas aeruginosa infection is unsure at this time, because he recently stepped on the nail, and there does not seem to be any signs of infection (erythema, warmth, discharge, etc). Careful follow-up is mandatory if he is not started on an antibiotic.

Advising him to soak his foot in an iodine solution at home (choice A) is not the most appropriate next step because this patient stepped on a nail and requires a Td toxoid at this time. Also, some believe that using an iodine solution does not prevent infection and may even delay healing.

A CT scan of his left foot (choice C) is unnecessary at this time. First of all, it appears as if the nail is intact and there is no foreign material left in his foot. But if you were to believe that some metal might be left in the wound, a plain x-ray can be used to show metallic material. A CT scan is used for plastic, wood, and other substances that are radiolucent.

It seems unnecessary to request an immediate orthopedic surgery consult (choice D) for this simple, shallow, clean puncture wound on this patient’s heel. If the wound were deep and dirty or if were through the metatarsal head (and the tennis sneaker), orthopedic surgery consultation should be sought. However, this patient still requires a Td toxoid at this time, even if you decide to call for the consult.

Sending him home with a prescription for an antibiotic that covers Pseudomonas aeruginosa (choice E) is inappropriate at this time because he requires a Td toxoid. The question as to whether or not you should prescribe an antibiotic for this clean wound, after administering the toxoid is a tough one. Technically, an antibiotic should be prescribed when signs of infection are present, but since Pseudomonas osteomyelitis can be devastating if it occurs, many people would probably prescribe an antibiotic to try to prevent an infection. In either case, this answer is still incorrect because you cannot send him home with an antibiotic without providing the Td toxoid.

 

  A 36-year-old man is brought to the emergency department after being extricated from a motor vehicle crash. He is brought in by ambulance and it is reported that he was a restrained passenger in a high-speed motor vehicle accident. He was conscious at the scene but his legs were pinned under the collapsed car. After being cut free, he was transported to the hospital. A rapid assessment reveals that the patient has no drug allergies and had not drunk alcohol prior to the crash. He is awake and alert with a Glasgow Coma score of 15/15. His temperature is 37.0 C (98.6 F), blood pressure is 160/100 mm Hg, pulse is 110/min, and respirations are 24/min. He denies pain in his neck on palpation and has full range of motion. Physical examination shows clear lungs, regular heart sounds, an open right humerus fracture, and bilateral lower extremity injuries. His left leg is intact but swollen and erythematous. The medical technicians report that the left leg was the pinned leg. His right leg appears to have an open femur fracture. He has 2+ radial pulses bilaterally. His left foot is cool compared with his right. The most ominous physical finding would be

  A. loss of deep tendon reflexes on the left
  B. loss of dorsalis or posterior tibial pulses
  C. pale color
  D. paraesthesias to touch
  E. tenderness on palpation
Explanation:

The correct answer is B. This patient likely has a compartment syndrome. Because portions of the body such as the leg have fascial compartments that are relatively noncompliant, tissue injury usually results in massive increases in pressure. Once the pressure is great enough to overcome arterial pulsations, ischemia and necrosis are imminent.

The loss of deep tendon reflexes on the left (choice A) is an early sign of compartment syndrome and in conditions such as gluteal compartment syndrome, is the most sensitive indicator of early compartment syndrome. Since it is the most sensitive finding, it does not portend imminent loss of tissue.

Pale color (choice C) and paraesthesias to touch (choice D) are also findings in compartment syndrome but are not as ominous as loss of pulses. Classically, of the six “Ps” of compartment syndrome: pallor, pain, paresthesia, pulselessness, poikilothermia, and paralysis, these two are the most nonspecific.

Because of the nature of the patient’s injuries, tenderness on palpation (choice E) is going to be present. Such a nonspecific finding is common to a majority of injuries and is not necessarily representative of an injury severe enough to cause a compartment syndrome.

 

A 53-year-old man who is status post renal transplant 3 years earlier, returns to the clinic for routine follow up for multiple warts on his hands and feet. Since his last visit 1 month ago he has been using large amounts of salicylic acid 40% plasters to treat his widespread warts. He tells you in passing that over the past week he has developed ringing in his ears. He currently takes many medications, including prednisone, cyclosporine, and lansoprazole. His temperature is 38.3 C (101.0 F), blood pressure is 110/80 mm Hg, and pulse is 105/min. Physical examination shows approximately 20 verrucous papules on the plantar surface of the feet as well as involvement of multiple fingers on the hands. The most appropriate next step in the management of this patient is to

  A. discontinue cyclosporine
  B. discontinue lansoprazole
  C. discontinue topical wart therapy
  D. refer him for an audiology exam
  E. tell him to continue current therapy until he sees an otolaryngologist about this problem
Explanation:

The correct answer is C. Discontinuing topical wart therapy is the correct answer because this patient is described to have widespread involvement of warts and covered each lesion with 40% salicylic plasters. Long-term systemic prednisone causes atrophy of skin, which results in increased absorption of topical salicylic acid. Salicylism commonly present with tinnitus, vomiting, tachycardia, and fever. Severe poisoning can result in seizures, coma, respiratory and cardiovascular failure, cerebral edema, and renal failure.

Cyclosporine (choice A) and lansoprazole (choice B) are both incorrect because neither medication typically result in tinnitus. In addition, there was no indication in the history that this patient has overdosed on his medication.

Referral to audiology (choice D) or otolaryngology (choice E) are both incorrect at this time. As mentioned above, salicylism can result in severe multiorgan failure if not caught early on. The first and most important step to take in this patient is to stop salicylic acid application. If tinnitus does not resolve, then a referral to a specialist is necessary.

 

A 69-year-old man with osteoarthritis and diabetes is seen in your office for a postoperative medical visit. He has been a long-time patient of yours and three days ago underwent a left total knee replacement. His hospital course was uneventful and he comes to the office today for a follow-up on his medical regimen. He continues to take his daily NPH insulin with good control of his blood sugars. He also continues to take the oxycodone for pain, that was given to him in the hospital. He is involved in a physical therapy rehabilitation program at the local hospital. The most immediate life-threatening risk to this patient in the perioperative period is

  A. addiction to pain medication
  B. a fall
  C. a pulmonary embolism
  D. renal failure
  E. a venous thrombosis
Explanation:

The correct answer is C. Orthopedic surgical procedures, especially those involving the hip and knee, have a very high incidence of intraoperative and postoperative deep venous thrombosis. The greatest risk to come from the presence of a DVT is fatal pulmonary embolism. Such events are, unfortunately, not rare.

Addiction to pain medication (choice A) is not a concern for this patient. There is a low probability that a postsurgical patient, requiring opiate medications for pain from that surgery, will become addicted to opiates. Belief in the contrary unfortunately is all too common in medicine and many patients are denied needed pain relief because of such irrational fears.

Falls (choice B) are always a concern with elderly patients, especially postoperatively. However, in quantifying specific risks, the consequences associated with a fall are much less than that of a pulmonary embolism.

Renal failure (choice D) is not likely at all in the near term. In the long term, it is very likely that her diabetes will result in some form of renal failure.

Although venous thrombosis (choice E) is very common postsurgically, this entity poses much less risk by itself, than a pulmonary embolism. A large number of these venous thrombosis occur in the non deep venous system and therefore pose very little, if any, risk to a patient.

 

  A 45-year-old woman is admitted to the hospital after suffering an inhalational burn injury. She was found in her living room by a fire-rescue team. She presented to the hospital with laryngeal edema and was intubated in the emergency department. The patient is now in the intensive care unit and is mechanically ventilated and sedated. Her vital signs are stable; her body temperature is now 37 C (98.6 F). After discussion with the team, it is decided that the patient will need escharotomy and skin grafting procedures over the ensuing 3 days. The decision is made to keep the patient sedated and paralyzed. The drug that is contraindicated in this patient is

  A. cis-atracurium
  B. d-curare
  C. diazepam
  D. morphine
  E. succinylcholine
Explanation:

The correct answer is E. Burn patients undergo a series of alterations in a variety of tissues. In muscle, there is a proliferation of non-junctional acetylcholine receptors. The consequence of this is that when a depolarizing muscle relaxant such as succinylcholine is used in these patients, the serum potassium may rise as much as 1.5 mEq/L. This may be enough to cause cardiac arrest and death.

Cis-atracurium (choice A) and d-curare (choice B) are non-depolarizing muscle relaxants that are safe in these patients. Owing to the increased numbers of acetylcholine receptors, the dosing of this class of drugs needs to be augmented.

Diazepam (choice C) is a benzodiazepine sedative that is also a mainstay of therapy for burn patients.

Morphine (choice D) is a mu-agonist opiate and is a mainstay of pain control in burn patients. Such patients typically require on average of 10mg per hour as opposed to post-surgical patients that may receive 10mg every 12 hours for pain control.

 

  A 68-year-old woman is brought to the emergency department after being rescued from her burning apartment building. She was found unconscious in her bedroom, which was full of smoke and flames. On presentation, she has extensive second and third degree burns over her legs and torso with an estimated TBSA burn of 25%. The patient is stable with a blood pressure of 160/70 mm Hg and a pulse of 100/min. The body temperature is 36 C (96.8 F). The plan is to initiate fluid replacement. The most appropriate management is to

  A. infuse 1ml/kg x TBSA % burn of colloid solution on post burn day 2 for 24 hours
  B. infuse 1ml/kg x TBSA % burn of colloid solution over 24 hours
  C. infuse 4ml/kg x TBSA % burn of lactated ringers solution over 8 hours
  D. infuse 4ml/kg x TBSA % burn of lactated ringers solution over 24 hours
  E. infuse 4ml/kg x TBSA % burn of lactated ringers solution with one half in the first 8 hours
Explanation:

The correct answer is E. Burn patients are acutely hypovolemic secondary to massive capillary leak and tissue sequestration of fluid. Many formulas for administering fluid resuscitation have been described, all in proportion to the %TBSA burned. One half of the calculated resuscitation volume for the first 24 hours is given in the first 8 hours. The formula in this answer is the Parkland formula, the most widely used resuscitation formula. Most centers use crystalloid (normal saline or lactated ringers solution) for volume resuscitation.

Infusing 1ml/kg x TBSA % burn of colloid solution on post burn day 2 for 24 hours (choice A) or 1ml/kg x TBSA % burn of colloid solution over 24 hours (choice B) are colloid based resuscitation formulas. Most authors in the Unites States believe the abundant evidence that outcome in burn patients is not influenced by early colloid administration. If colloid has any role in the management of acute thermal injury it is after the initial 24-hour post-burn period.

Infusing 4ml/kg x TBSA % burn of lactated ringers solution over 8 hours (choice C) and over 24 hours (choice D) are incorrect because of the duration of infusions.

 

A 30-year-old man is brought to the emergency department following a motor vehicle accident. He has no medical history, is on no medications, and has no allergies to any medications. You are told that he was a restrained front passenger in a head-on collision. He was observed to aspirate on the scene and was intubated by EMTs en-route for respiratory distress. He is intubated, sedated, and has a cervical collar in place. His temperature is 37.2 C (99 F), blood pressure is 110/65 mm Hg, pulse is 110/min, and respiratory rate is 23/min. His oxygenation is 100% on 100% oxygen. His has decreased breath sounds over the right lower lung field and cardiac exam is significant for regular tachycardia. A chest radiograph shows a right lower lobe consolidation. The most appropriate next step in the management of his aspiration is

  A. close monitoring with supportive care
  B. administration of intravenous antibiotics
  C. administration of intravenous steroids
  D. administration of nebulized acetylcysteine
  E. administration of nebulized beta agonists
Explanation:

The correct answer is A. Aspiration typically causes a 2-staged disease process. In the immediate stage, it causes a chemical pneumonitis which can present with consolidation on chest radiograph, fever, and/or leukocytosis. This pneumonitis can resolve without medical intervention. Aspiration pneumonia typically presents as a delayed complication and is typically seen in patients who are at a risk of having nonsterile gastric contents (diabetic gastroparesis, small bowel obstruction, etc.). Not all patients with a pneumonitis will go on to develop pneumonia. Therefore, in this patient it is most appropriate to institute supportive care and monitor for the subsequent development of pneumonia. This will decrease the risk of complications from indiscriminate use of antibiotics such as selecting a resistant organism.

Administration of intravenous antibiotics (choice B) is not appropriate in this acute setting since this patient most likely has an aspiration pneumonitis and not an aspiration pneumonia as discussed above.

Administration of intravenous steroids (choice C) has not been shown to have any clinical benefit in patients with aspiration, even in those with the aspiration pneumonitis.

Administration of nebulized acetylcysteine (choice D), a mucolytic agent, has not role in the typical management of aspiration. This agent can be used in patients with respiratory difficulty from thick mucous plugs, such as with cystic fibrosis.

Administration of nebulized beta agonists (choice E) has not been shown to have any benefit following aspiration in patients who are not bronchospastic.

 

A 28-year-old woman comes to the office because of a 3-day history of a foul smelling vaginal discharge. She has no other complaints. Pelvic examination shows a yellowish-green frothy discharge, edema of the vulva, and petechiae on the cervix. A wet mount of the discharge shows motile, flagellated organisms. You prescribe the appropriate therapy in a single dose and she reminds you that she is breast-feeding her 3 month old infant. At this time you should

  A. advise her to discontinue breast-feeding indefinitely
  B. prescribe a different pharmacologic agent
  C. recommend that she pump and discard her milk for 24 hours and then resume breast-feeding
  D. take back the prescription and tell her that treatment of her infection is not necessary
  E. tell her that this medication does not appear in breast milk
Explanation:

The correct answer is C. This patient has an infection with Trichomonas vaginalis, and should be treated with metronidazole. Metronidazole appears in breast milk, and the effect on a breast-fed infant is unknown but may be of concern. Therefore, the American Academy of Pediatrics recommends that the mother be given a single 2-g dose of metronidazole and then that she pump and discard her milk for 24 hours. Then she can resume breast feeding.

It is inappropriate to advise her to discontinue breast-feeding indefinitely (choice A). The medication will not remain in her milk “indefinitely”, and therefore the recommendation is to discontinue breast-feeding for 12-24 hours after a single 2- gram dose of metronidazole or to pump and discard her milk for 24 hours, and then resume breast-feeding.

Metronidazole is the treatment of choice for Trichomonas vaginalis. It is unnecessary to prescribe a different pharmacologic agent (choice B) because all the patient needs to do is discontinue breast-feeding for 12-24 hours after a single 2- gram dose of metronidazole or to pump and discard her milk for 24 hours and then resume breast-feeding.

It is inappropriate to take back the prescription, and tell her that treatment of her infection is not necessary (choice D), because this patient has a symptomatic Trichomonas vaginalis infection that should be treated appropriately with metronidazole.

Since metronidazole does appear in breast milk, it is incorrect to tell her that it doesn’t (choice E).

 

A 72-year-old woman comes to the clinic because of a new onset of terrible headaches, accompanied by pain in her left eye. She complains of seeing “halos” and is very nauseated. Her vision is blurry. On the examination, her eye is hard and tender to touch and the pupil is dilated and midfixed. She has no previous history of similar ophthalmologic problems. Her previous history is significant for chronic lower back pain after an injury in a motor vehicle accident years ago. She was recently started on new medication by her neurologist to target insomnia because of the back pain and the pain itself. To avoid further problems, the medication that should be discontinued, that is most likely causing her current ophthalmologic problems is

  A. amitriptyline
  B. carbamazepine
  C. gabapentin
  D. propranolol
  E. trazodone
Explanation:

The correct answer is A. Amitriptyline is a tricyclic antidepressant with the strongest anticholinergic and sedative properties in its group. It is frequently used in low doses for the treatment of chronic pain. Its strong anticholinergic effects can cause constipation, urinary retention, dry mouth, acute glaucoma, paralytic ileus, etc.

Carbamazepine (choice B) is an anticonvulsant also used in the treatment of neuralgic pain. It doesn’t cause anticholinergic side effects, but it can have effects on liver enzymes and bone marrow.

Gabapentin (choice C) is an anticonvulsant used as an adjunctive in the treatment of pain. It has no anticholinergic side effects that would cause glaucoma.

Propranolol (choice D) is a beta blocker that is not used in the treatment of chronic pain. It is actually used to treat a glaucoma attack, and thus should not be discontinued.

Trazodone (choice E) is an antidepressant with serotonergic properties. It is used for the treatment of insomnia in low doses. Its main side effects include orthostatic hypotension and sedation. It is not used in the treatment of pain.

 

 

A 45-year-old man is struck by a motor vehicle and is transported by a med flight to the local emergency department. He is reported to be intoxicated with a Glasgow score of 12/15 on the scene of the accident. He was struck by a vehicle while crossing the street. On arrival, the patient is awake and somewhat combative. He is alert to person only. His temperature is 37.0 C (98.6 F), blood pressure is 150/80 mm Hg, pulse is 112/min, and respirations are 20/min. Primary survey reveals a well-developed man in mild distress. He is in a cervical collar. He has obvious lower extremity tibia fractures bilaterally and a laceration on his forehead. The most appropriate manner to clear the patient’s cervical spine from injury is

  A. cervical spine plain radiographs
  B. CT scan of the neck
  C. flexion, extension, and lateral rotation of the neck while assessing any pain response from the patient
  D. manual palpation of the patient’s neck for any obvious injury
  E. this patient cannot have his cervical spine cleared at this time
Explanation:

The correct answer is E. This patient is at high risk for a cervical spine injury and a key component of any trauma evaluation is ensuring that none exists in situations such as these. The two components of a successful evaluation for clearance are an imaging test of some sort and a lucid, mentally clear patient. Once imaging is completed, the patient is asked, during the examination part, to express pain or discomfort if there is any. This patient is intoxicated and therefore is not able to reliably indicate pain during the examination. Once he is sober the exam can be undertaken once his imaging study is read as normal.

Cervical spine plain radiographs (choice A) or CT scan of the neck (choice B) are both appropriate selections for imaging. Depending on the expertise of the radiologist, the sensitivity of either for detecting cervical spine injury is equivalent. No imaging mode however can be utilized in the absence of a physical examination.

Flexion, extension, and lateral rotation of the neck while assessing any pain response from the patient (choice C) would be the appropriate sequence of events after a cleared image for any obvious injury if he were not intoxicated.

Manual palpation of the patient’s neck for any obvious injury (choice D) is not an adequate physical examination. Exam must include lateral rotation and flexion-extension of the patient’s neck as well.

 

A 45-year-old man is struck by a motor vehicle and is transported by a med flight to the local emergency department. He is reported to be intoxicated with a Glasgow score of 12/15 on the scene of the accident. He was struck by a vehicle while crossing the street. On arrival, the patient is awake and somewhat combative. He is alert to person only. His temperature is 37.0 C (98.6 F), blood pressure is 150/80 mm Hg, pulse is 112/min, and respirations are 20/min. Primary survey reveals a well-developed man in mild distress. He is in a cervical collar. He has obvious lower extremity tibia fractures bilaterally and a laceration on his forehead. The most appropriate manner to clear the patient’s cervical spine from injury is

  A. cervical spine plain radiographs
  B. CT scan of the neck
  C. flexion, extension, and lateral rotation of the neck while assessing any pain response from the patient
  D. manual palpation of the patient’s neck for any obvious injury
  E. this patient cannot have his cervical spine cleared at this time
Explanation:

The correct answer is E. This patient is at high risk for a cervical spine injury and a key component of any trauma evaluation is ensuring that none exists in situations such as these. The two components of a successful evaluation for clearance are an imaging test of some sort and a lucid, mentally clear patient. Once imaging is completed, the patient is asked, during the examination part, to express pain or discomfort if there is any. This patient is intoxicated and therefore is not able to reliably indicate pain during the examination. Once he is sober the exam can be undertaken once his imaging study is read as normal.

Cervical spine plain radiographs (choice A) or CT scan of the neck (choice B) are both appropriate selections for imaging. Depending on the expertise of the radiologist, the sensitivity of either for detecting cervical spine injury is equivalent. No imaging mode however can be utilized in the absence of a physical examination.

Flexion, extension, and lateral rotation of the neck while assessing any pain response from the patient (choice C) would be the appropriate sequence of events after a cleared image for any obvious injury if he were not intoxicated.

Manual palpation of the patient’s neck for any obvious injury (choice D) is not an adequate physical examination. Exam must include lateral rotation and flexion-extension of the patient’s neck as well.

 

  A 40-year-old woman is admitted to the hospital because of fever, jaundice, and abdominal pain. Laboratory studies on admission showed only an elevated leukocyte count and an ultrasound of the abdomen showed dilated bile ducts. An endoscopic retrograde cholangiopancreatogram (ERCP) was performed and a stone was extracted. Today, she had 3 episodes of vomiting and is complaining of “deep” epigastric pain that radiates to the back, is worsened by lying flat on her back, and somewhat relieved by leaning forward. Her skin is cool and clammy and her abdomen is distended and tender. Laboratory studies show a leukocyte count of 21,000/mm3, an amylase level of 450 U/L, and a lipase level of 400 U/L. She underwent a laparoscopic appendectomy 3 years earlier. The most likely explanation for the worsening of her symptoms is

  A. erosion of the gastroduodenal artery
  B. inflammation of the pancreas caused by endoscopic retrograde cholangiopancreatography
  C. intestinal obstruction caused by previous surgery
  D. passage of a gallstone into the duodenum (gallstone ileus)
  E. surreptitious alcohol consumption
Explanation:

The correct answer is B. This patient has developed acute pancreatitis following the ERCP. This is a known complication of this procedure, which is used to assess the cause, location, and extent of biliary obstruction and to remove the stone. It is thought to be caused by injury to the ampulla of Vater and/or retrograde injection of contrast material. Other complications of this procedure include cholangitis, infected pancreatic pseudocyst formation, and perforation.

Erosion of the gastroduodenal artery (choice A) is an unlikely cause of the worsening of this patient’s symptoms because this is typically associated with a duodenal ulcer. The patient will often present with symptoms of a duodenal ulcer (gnawing epigastric pain that is relieved by food) and then the sudden worsening of symptoms and gastrointestinal bleeding. A duodenal ulcer on the posterior wall may also penetrate through the pancreas, liver, or biliary tree and may lead to increased pain and elevated amylase levels. However, the patient in this case had a gallstone in a bile duct, not a duodenal ulcer.

Intestinal obstruction caused by previous surgery (choice C) is an unlikely cause for the worsening of her symptoms. Even though an intestinal obstruction may be associated with elevated amylase levels, this patient’s prior appendectomy is less likely to be causing these symptoms than the recent ERCP. Intestinal obstruction is characterized by crampy abdominal pain, vomiting, obstipation, and abdominal distention. Also, the description of the pain that it is worsened by lying down and somewhat relieved by leaning forward is more consistent with pancreatitis than obstruction.

The passage of a gallstone into the duodenum (choice D) is associated with a gallstone ileus and typically presents with symptoms of intestinal obstruction (crampy abdominal pain, vomiting, obstipation, and distention). It is usually associated with cholecystitis and fistula formation. This patient’s presentation is more consistent with pancreatitis than gallstone ileus.

While alcohol consumption (choice E) is one of the most common causes of pancreatitis, it is very unlikely that this patient has been surreptitiously consuming alcohol and there is no evidence presented in the case that would suggest alcohol consumption. Therefore, it is more likely that her pancreatitis is due to the ERCP than to alcohol.

 

  A 47-year-old man comes to the office for a periodic health maintenance examination. You have been treating him over the years for hypertension and diabetes mellitus with enalapril and insulin. Lately you have been following his cholesterol, which has been elevated despite an attempt at a low-fat diet, a new moderate exercise program, and the cessation of smoking. In discussing this issue with him, you decide that considering all of his other risk factors for heart disease, it is time to start him on lovastatin. You explain the risks and benefits associated with this drug, and you give him the prescription before he leaves the office with instructions to call your office immediately if he experiences any of the common side effects. Three weeks later, you receive a frantic message from your answering service that this patient has called to tell you that he has side effects from the medication. The complaint that you expect to hear about when you return his call is

  A. constipation
  B. flushing
  C. insomnia
  D. muscle aches
  E. nausea
Explanation:

The correct answer is D. Myositis and abnormal liver function are the two main side effects associated with HMG-CoA reductase inhibitors, which are one of the most common classes of drugs prescribed for elevated cholesterol. While these side effects are relatively uncommon (1-2%), they can be very serious and have led to fatalities. Myositis occurs more frequently when combined with other cholesterol lowering agents. Discontinuation of the medication is important if these symptoms occur.

Constipation (choice A) is associated with bile-acid resins, such as cholestyramine.

Flushing (choice B) is associated with niacin, which is effective in decreasing the synthesis of VLDL and LDL. Other side effects include tachycardia, pruritus, nausea, diarrhea, elevated uric acid levels, and impaired glucose tolerance.

Insomnia (choice C) is not commonly associated with any of the commonly prescribed cholesterol-lowing agents. It is associated with methylphenidate, which is prescribed for attention-deficit hyperactivity disorder.

Nausea (choice E) is associated with niacin, cholestyramine, and fibric acid derivatives, such as gemfibrozil. It is not commonly associated with HMG-CoA reductase inhibitors.

 

 

A 26-year-old man is admitted to the hospital after accidental ingestion of corrosive alkali liquid. He denies any past medical or surgical history. The day before admission, he was at a party with his friends where he consumed a lot of alcohol. At the end of the party, he returned home and accidentally ingested corrosive alkali liquid from a bottle, mistaking it for a bottle of water. At the time of admission to the hospital, he complained of substernal chest pain and that he was feeling “really ill”. Initial gastrointestinal contrast study with water-soluble contrast did not reveal a gastrointestinal leak or perforation. His vitals are stable with a tachycardia of 90/min. The next best step in management is to

  A. continue to observe him in the hospital
  B. order a CT scan of the chest
  C. order an electrocardiogram
  D. order an upper gastrointestinal study with barium
  E. perform an upper gastrointestinal endoscopy
Explanation:

The correct answer is D. Chest pain, fever, tachycardia, subcutaneous emphysema, dysphagia, and dyspnea are suggestive of an esophageal perforation. An esophageal perforation may result from endoscopic procedures, external trauma, esophageal disease, and spontaneous perforation from violent bouts of emesis. Prompt recognition of an esophageal perforation is necessary to prevent delayed complications. Whenever a perforation is suspected, a contrast study should be performed with water-soluble contrast material. If this study does not demonstrate the perforation, it should be repeated with barium. Barium is more accurate for a delineating esophageal leakage. Contrast studies not only help in diagnosing esophageal rupture but also document the level of injury, which has important implications for treatment.

Early diagnosis of esophageal perforation is necessary. Continued observation (choice A) of a suspected esophageal perforation may lead to continued leakage and sepsis. Results of esophageal perforation repair are better if the perforation is detected early.

A CT scan of the chest (choice B) is not sensitive enough as the first investigation in a suspected esophageal perforation. The contrast leak from perforation may show up on the CT scan. Pleural effusions may provide indirect evidence of esophageal perforation.

An electrocardiogram (choice C) is not indicated in a young patient with a suspected esophageal perforation.

An upper gastrointestinal endoscopy (choice E) is contraindicated as the first investigational choice in a suspected perforated esophagus. Rigid esophagoscopy may be indicated to extract the foreign body from esophagus.

 

 

An unconscious 21-year-old man is rushed into your emergency department following a high speed motor vehicle accident. He has bilateral upper extremity fractures and no obvious lower extremity fractures. While your team members manage his airway and stabilize his neck, you are given the responsibility of obtaining vascular access. His blood pressure is 105/54 mm Hg and pulse is 118 /min. Your line should be placed

  A. above the inguinal ligament where the femoral vein crosses the femoral artery
  B. in the femoral artery lying lateral to the femoral vein
  C. in the femoral artery lying medial to the femoral vein
  D. in the femoral vein lying lateral to the femoral artery
  E. in the femoral vein lying lateral to the femoral nerve
  F. in the femoral vein lying medial to the femoral artery
  G. in the internal jugular vein
Explanation:

The correct choice is F. This question tests your knowledge of two topics, management of the trauma patient and knowledge of the anatomy of the femoral canal. Knowledge of the anatomy of the femoral canal will allow for appropriate placement of a large bore catheter into the venous system. From lateral to medial, the anatomy of the femoral canal is femoral nerve, femoral artery, the femoral vein, and lymphatic channels. The only answer that meets all of the necessary criteria is in the femoral vein lying medial to the femoral artery. An unstable trauma patient requires venous access with a large bore catheter that has multiple ports to allow for the administration of fluids, blood products, and medications simultaneously. In this patient the catheter should be placed in the groin.

Above the inguinal ligament (choice A) the femoral artery and vein are named the external iliac artery and vein.

Arterial access (choices B and C) will not allow for the delivery of medications, fluids, and blood products directly to the heart for systemic distribution.

In the femoral vein lying lateral to the femoral artery (choice D) is incorrect because as stated above, the femoral vein is medial to the femoral artery.

In the femoral vein lying lateral to the femoral nerve (choice E) is incorrect because as stated above, the femoral vein is medial to the femoral artery.

The patient’s jugular vein (choice G) will be difficult to access as his neck requires stabilization in a collar until appropriate radiographic studies are obtained. Upper extremity access is limited secondary to bilateral fractures.

 

 

  A 72-year-old man returns to the office following an esophagoscopy done earlier today because of a progressive dysphagia for the past 6 months. A 0.8-cm ulcerated lesion is found and a biopsy is taken. He is now, 4 hours later, complaining of severe chest pain. His temperature is 38.3 C (101.0 F), blood pressure is 130/80 mm Hg, pulse is 125/min, and respirations are 28/min. Physical examination shows crepitation in the neck and a crunching sound over the heart. You call an ambulance and escort him to the hospital. After you obtain a surgical consult, the most appropriate next step in management is to

  A. obtain an immediate bronchogram
  B. order an immediate esophagogram with a water-soluble agent
  C. perform an immediate bronchoscopy
  D. perform an immediate thoracotomy
  E. repeat the esophagoscopy to evaluate the biopsy site
Explanation:

The correct answer is B. This patient most likely has an esophageal perforation, which is a serious, life-threatening complication of esophagoscopy. The signs and symptoms may include chest pain near the rupture site, fever, crepitus, and air in the mediastinum behind the heart. An immediate esophagogram with a water-soluble agent is indicated. The treatment is wide drainage by thoracotomy.

A bronchogram (choice A) is used to define the bronchial anatomy, and is used to evaluate a patient’s bronchiectasis. It is not indicated in a case of esophageal perforation.

Bronchoscopy (choice C) is used to evaluate a patient with a suspected bronchial obstruction or endobronchial disease. It is not indicated in this patient with a suspected esophageal perforation.

Thoracotomy (choice D) is the treatment for an esophageal perforation, however an immediate esophagogram with a water-soluble agent should be performed to establish the diagnosis.

Repeating the esophagoscopy to evaluate the biopsy site (choice E) is not the preferred method to identify an esophageal perforation. An esophagogram is better to evaluate for a suspected perforation.

 

 

A 15-year-old boy is brought into your office by his parents because of numbness and tingling in his feet, slowly increasing over the last several weeks. He has attention deficit hyperactivity disorder for which he has been taking methylphenidate for the past 4 years. In addition, he had a positive PPD skin test after screening upon entrance to high school and was placed on isoniazid 6 months ago. He takes no other medications and has no allergies. His parents feel that he has become more difficult to control since entering high school and the father confides to you that he suspects these “latest complaints” are a method of avoiding physical education class. He answers your questions readily and reports that gym is actually his favorite class at school. He denies any problems at home, with friends, or at school. On physical examination, you find nothing abnormal except hyperesthesia over a stocking distribution of both lower extremities. Deep tendon reflexes, motor function, and sensation are intact. This patient’s current condition would have most likely been prevented by supplementation with

  A. folate
  B. pyridoxine
  C. riboflavin
  D. thiamine
  E. vitamin E
Explanation:

The correct answer is B. One of the side effects of isoniazid therapy is peripheral neuropathy, which occurs more frequently in children. Supplementation with pyridoxine, 25-50 mg PO qd, is commonly used to decrease the incidence of neuropathy. Pyridoxine is also known as vitamin B6.

Folate (choice A), riboflavin (choice C), thiamine (choice D), and vitamin E (choice E), are not helpful in preventing this adverse effect.

 

Seven days ago, a 62-year-old man underwent a colonic resection for carcinoma. His postoperative stay is prolonged because of pneumonia. His past history is significant for insulin-dependent diabetes mellitus, hypertension, congestive heart failure, and bronchitis. He has a 30-pack year smoking history. Colonic resection was uneventful. He is tolerating a regular diet and is scheduled to be discharged home the next day. You are called by the nurse to check his wound as she noticed a new serous discharge. His temperature is 37.0 C (98.6 F), pulse is 80/min, blood pressure is 130/80 mm Hg, and blood glucose by finger stick is 240 mg/dL. Abdomen is soft and non-tender. There is no induration, erythema, or crepitus of the wound. Copious amounts of serous brownish liquid is expressed from the wound on gentle pressure. Fluid is expressed spontaneously on coughing from both upper and lower parts of the wound. The most appropriate next step is to

  A. control his blood sugar
  B. order diuretics
  C. order dry dressings to be continued
  D. prescribe a 1-week course of antibiotics
  E. prepare the patient for another laparotomy
Explanation:

The correct answer is E. Abdominal wound dehiscence due to poor facial healing occurs typically between 7-10 days after laparotomy. Patients with chronic bronchitis, diabetes, and a smoking history, are prone to poor facial healing. Poor wound closure technique is the main cause of wound dehiscence. New onset serous discharge from a laparotomy wound should raise the suspicion of wound dehiscence. Facial integrity should be checked when dehiscence is suspected. The pouring of fluid from both upper and lower parts of the wound should lead you to suspect complete dehiscence, which needs to be fixed by a re-laparotomy.

Control of blood sugar is essential in a diabetic for better wound healing, but dehiscence of the wound requires re-laparotomy and suturing of the wound, not just controlling the blood sugar (choice A).

Congestive heart failure causes edema of the lower extremities and pulmonary congestion. Serous discharge from the wound is not a sign of congestive heart failure. Diuretics are not helpful in the healing of the wound (choice B).

Simple application of dressings to the dehisced wound would not prevent complete dehiscence and evisceration. Dry dressings (choice C) are not an option in treating complete wound dehiscence.

Antibiotics are indicated in wound infection, but not in wound dehiscence (choice D).

 

 

  A 47-year-old man is brought into the emergency department by ambulance after a motor vehicle accident. He is conscious, alert and, besides the lacerations on his scalp, he seems to be otherwise well. His neck is stabilized by a brace and he asks you to remove it. You assess his neurologic exam to be non-focal, but you obtain cervical neck films anyway. No cervical spine abnormalities are noted and you remove the cervical collar. Suddenly, the patient’s pulse rises to 130/min, his blood pressure drops to 80/50 mm Hg, and his respiratory rate increases to 35/min. His physical examination reveals jugular venous distension, peripheral and perioral cyanosis, absent breath sounds on the left side of his chest, a tachycardic but otherwise normal cardiac examination, and a tender abdomen in the left upper quadrant. The most appropriate initial step is to

  A. insert a needle thoracostomy into the second intercostal space
  B. insert a tube thoracostomy into the pleural space
  C. intubate the patient for hypoxemic respiratory failure
  D. order a portable chest x-ray
  E. perform a diagnostic peritoneal lavage for a splenic rupture
Explanation:

The correct answer is A. This patient presented with the classic signs and symptoms of a tension pneumothorax, which presents a unilateral loss of breath sounds with hypertympany, shift of the trachea away from the injured side, and jugular venous distension. The diagnosis is based upon a characteristic history and examination. He requires a needle thoracostomy into the second intercostal space.

The patient may require a tube thoracostomy (choice B) for his tension pneumothorax; however a needle thoracostomy to decompress his lung would be the first step prior to insertion of a chest tube.

Intubating this patient (choice C) may be required if his hypoxemia doesn’t resolve with a needle thoracostomy and supplemental oxygen. However, it would be premature as an initial first step

Instead of a diagnostic test such as a chest x-ray (choice D), a tension pneumothorax needs to be emergently treated. Decompression is accomplished by inserting a large-bore needle into the second intercostal space in the midclavicular line on the affected side.

A diagnostic peritoneal lavage (choice E) may be required to evaluate the patient’s abdominal tenderness. His hypotension and tachycardia could be secondary to acute blood loss, especially from an intraabdominal source like a splenic rupture. However, airway and breathing must be evaluated and treated prior to circulation.

 

 

A 24-year-old third-year medical student is brought to the emergency department from the operating room after she began to have trouble breathing after she put on her gown, latex gloves, and mask. This was going to be her first time “scrubbing in” to a case on the first day of her surgery rotation. Another medical student who was “scrubbed in” to the case, tells you that the she was grasping at her throat and attempting to rip off her gown. She is now on a stretcher, gasping for air. She is brought into a room and all of her clothes are taken off, revealing red hands and an eruption of well-circumscribed, erythematous, raised lesions covering her entire body. Her blood pressure is 70/50 mm Hg, pulse is 110/min, and respirations are 45/min. She has marked laryngeal edema and audible wheezes. The other student says that this is the second day of their third year, and that the only other patient contact that they have had was during the first year when they went as a group to evaluate a patient in respiratory isolation. At this time the most correct statement about her condition is:

  A. Her rash is unrelated to her respiratory symptoms
  B. Her symptoms are caused by the anxiety and stress from the first surgical experience
  C. Her symptoms are due to an infectious pathogen
  D. Her symptoms are the result of bioactive mediators released when exposed to an antigen
  E. She should be given indomethacin immediately to reduce the respiratory symptoms
Explanation:

The correct answer is D. This patient is experiencing an anaphylactic reaction, which is a life-threatening response in a sensitized individual to a specific antigen, most likely the latex in the gloves, and is associated with the release of lipid mediators, secretory granule preformed mediators, and cytokines. The manifestations typically occur within seconds to minutes of exposure and include respiratory distress, vascular collapse, urticaria and angioedema, gastrointestinal symptoms, and shock. The reaction involves IgE-dependent activation of mast cells, basophils and the release of mediators such as histamine, cytokines, and lipid mediators. The immediate treatment involves the ABCs (intubation, oxygen, and intravenous saline), epinephrine, a vasopressor agent such as dopamine, an antihistamine such as diphenhydramine, and glucocorticoids to alleviate later recurrence of symptoms.

It is incorrect to say that the rash is unrelated to her respiratory symptoms (choice A) because they are both associated anaphylactic reactions. Anxiety and stress from the first surgical experience (choice B) may be associated with vasovagal (vasodepressor) syncope, which is the cause of the common “faint”. It frequently occurs during periods of stress or fear and often occurs in medical students during new experiences. The symptoms include hypotension, bradycardia, nausea, pallor, diaphoresis, and a sudden, transient loss of consciousness. Lying down and elevating the legs reverses the symptoms. This is very different from a life-threatening anaphylactic reaction where hypotension, tachycardia, respiratory failure, shock, and skin manifestations occur. Assuming a supine position during an anaphylactic reaction does not reverse the symptoms.

This patient is most likely experiencing an anaphylactic reaction from the latex gloves, not an infectious pathogen (choice C). Pathogens may cause septic shock by releasing toxins that then activate cytokines, platelet activating factors, arachidonic acid metabolites, and humoral defense systems. This manifests as hypotension, tachycardia, changes in mental status, and cold extremities. The laryngeal edema and specific rash that this patient has are more consistent with an anaphylactic reaction than septic shock.

She should definitely not be given indomethacin immediately to reduce the respiratory symptoms (choice E) because it is a nonsteroidal anti-inflammatory drug that may cause similar life-threatening respiratory reactions in patients with asthma. You do not know her medical history. Also, this is not part of the management of anaphylaxis, which includes epinephrine, saline, vasopressors, and intubation.

 

 

A 15-year-old boy is brought to the office by his father for a physical examination before summer camp. Since baseball season is in full swing, and he is the pitcher on the varsity team, he comes to the office in uniform to be able to go straight to the game after the appointment. He talks incessantly about the game and his father chimes in every so often to show his support for his son. During the examination, the father talks non-stop about getting his son a full baseball scholarship to an ivy league college. He has been in touch with all of the coaches at the schools. You are surprised to see how muscular the boy has become since his physical examination the previous year. You inquire about weight-lifting, and he says that he works-out in their basement daily. His father bought him a full weight set for his birthday. You suspect that he is taking anabolic steroids. During the evaluation, particular attention should be given to

  A. funduscopic examination
  B. neurologic reflexes
  C. respiratory examination
  D. testicular size
  E. thyroid examination
Explanation:

The correct answer is D. Testicular atrophy may be seen with anabolic steroid usage, which is common in athletes trying to increase muscle mass and strength, and maintain a “winning edge”. Athletes may use anabolic steroids in very high doses that lead to many adverse effects that may include liver disease, gynecomastia, impotence, acne, striae, edema, increased cholesterol, aggression, and depression.

Anabolic steroids are not typically associated with abnormalities in the funduscopic examination (choice A), neurologic reflexes (choice B), respiratory examination (choice C), and thyroid examination (choice E).

 

 

  A 78-year-old woman with non-insulin dependent diabetes mellitus is admitted to the hospital because of vomiting, abdominal pain, and obstipation. Her past surgical history is significant for appendectomy, hysterectomy, and a cholecystectomy. She was found at home very dehydrated and vomiting. On admission she is lethargic. Her blood pressure is 100/60 mm Hg and pulse is 100/min. Her mucous membranes are dry, her abdomen is distended, but there are no peritoneal signs. An incarcerated hernia is found and she undergoes a laparotomy. At laparotomy, a bowel resection is performed for ischemic bowel segment and the hernia is repaired. She received 100 cc of intravenous fluids per hour postoperatively. On the third postoperative day, she spiked a fever of 39.1 C (102.4 F). Physical examination reveals dry oral mucous membrane and a tender, 3 cm nodule in front of the right ear. The most appropriate next step in the management is to

  A. begin aggressive hydration
  B. begin appropriate oral hygiene
  C. prepare her for immediate surgical drainage
  D. send blood cultures and await sensitivities
  E. start empiric therapy for Staphylococcus aureus
Explanation:

The correct answer is E. Acute suppurative parotitis is a severe, life threatening infection of the parotid gland, most often seen in elderly or debilitated patients who are severely dehydrated. Pathogenesis is thought to be related to stasis within salivary ducts as a result of increased viscosity. Staphylococcus aureus is the organism most often found in this severe infection. Initial treatment with appropriate intravenous hydration, sialogogues, and antibiotics against Staphylococcus may be successful. If improvement is not seen within 12 hours of initiating this treatment, surgical drainage is warranted.

Aggressive hydration (choice A) is essential to correct the dehydration in this elderly patient, although treatment of acute suppurative parotitis should be started at the time.

Oral hygiene (choice B) is not the main therapy for acute suppurative parotitis.

Immediate surgical drainage (choice C) is not indicated unless the patient does not improve in 12 hours.

Blood cultures (choice D) are essential to isolate the causative organism, but empiric antibiotic therapy should not be delayed awaiting the sensitivities.

 

 

A 6-year-old girl is brought to the emergency department 10 hours after being bitten on her face while she was playing “dog” with her friend. The two girls were on the ground with the family dog, laughing, growling, and “biting the air,” when all of a sudden the patient felt a “sharp, stabbing pain” on her right check. She ran to the mirror and started screaming when she saw blood on her face. “It all happened so fast” that nobody is exactly sure what happened. Both the friend and the dog had blood on their mouths, but they are unsure if it was the girl or the dog who actually broke the patient’s skin. The dog and both girls are up-to-date with all vaccinations. Physical examination shows a 2-cm laceration on her right check. The surrounding tissue is tender. You request a plastic surgery consult after culturing and copiously irrigating the wound. The most appropriate additional treatment is to

  A. administer ampicillin/sulbactam, intravenously
  B. administer metronidazole, intravenously
  C. give her clindamycin, orally
  D. give her penicillin V, orally
  E. provide no antibiotic therapy at this time
Explanation:

The correct answer is A. This patient has a bite wound on her face that was either caused by a dog or another human, and she requires antibiotic prophylaxis. The most appropriate of all of the choices is ampicillin/sulbactam, intravenously because it covers the most likely pathogens such as S. aureus, P. multocida, H. influenzae, and B-lactamase-positive oral anaerobes. This girl requires treatment because all human bites require antibiotics, and dog bites to the face typically require treatment.

Metronidazole (choice B) is not the preferred antibiotic prophylaxis for human or dog bites. It is given for anaerobic infections, giardiasis, trichomonas, pseudomembranous colitis, and bacterial vaginosis.

Clindamycin (choice C) may be given to cover Gram-positive cocci and anaerobes. However, it is best given along with trimethoprim-sulfamethoxazole to cover the other potential pathogens. However, this is not the preferred antibiotic regimen. Since there is a chance that she will be admitted to the hospital and because it is a facial wound that might have been inflicted by another person, an intravenous route of administration is best (even if it is one dose in the emergency department and then the patient is discharged with a prescription for oral therapy).

Penicillin V, orally (choice D) is not the preferred antibiotic prophylaxis for human or dog bites because of poor coverage of some anaerobes and Gram-negative organisms, and most of the Gram-positive organisms present in human or dog bite infections. Penicillin V is typically used to treat streptococcal infections.

No antibiotic therapy is indicated (choice E) in non-extensive dog bites that do not appear infected, do not occur on the face or involve a bone or joint, and when there are no co-morbidities. Human bites always require antibiotics and often require hospital admission. Since they are unsure as to the source of the bite, and it involved the face, antibiotics are indicated.

 

 

  A 24-year-old man returns to your office 10 days after a right knee arthroscopy for a medial meniscus tear. The patient originally injured his knee playing football with his brothers and an MRI disclosed a meniscal tear. The patient has no other medical history and underwent a successful arthroscopic meniscus removal under general anesthesia. He reports that since his surgery his voice has been hoarse and he often feels liquid get into his “windpipe”. He should be told that:

  A. He likely has an upper respiratory tract infection
  B. He should have his vocal cords evaluated by a otolaryngologist
  C. His symptoms are caused by a thyroid problem
  D. His vocal cords were damaged from the endotracheal tube
  E. It is normal to have a sore throat and hoarseness after surgery
Explanation:

The correct answer is B. One of the risks of general anesthesia is damage to the vocal cords by direct trauma. Although rare, it does occur and it manifests itself in just the way this patient presents; hoarseness after surgery that does not improve after one week. Any such patient should have a full ENT evaluation to determine if there is any cord paralysis or paresis.

This patient has no evidence of any upper respiratory infection (choice A). Even if he did however, given his proximity to general anesthesia and an endotracheal tube, it is much more likely that his symptoms are due to this event and not a URI.

Given that the patient has recently had a general anesthetic, the likelihood that there is a thyroid problem (choice C) responsible for his hoarseness is very small.

Although it is possible that his vocal cords were damaged from the endotracheal tube (choice D), this has yet to be established by physical examination of the cords. Although likely, making the definitive diagnosis and failing to visualize the cords could result in a missed diagnosis of laryngeal polyp or other disturbance causing the hoarseness.

Although it is normal to have a sore throat and hoarseness after surgery (choice E), this is only true in the immediate postoperative period of less than 24 hours. There is nothing normal about residual hoarseness at 10 days postoperatively.

 

 

  The mother of a 3-year-old patient of yours, calls the office after the child banged his head against the coffee table in their family room. The mother is absolutely frantic, saying that her son has been “screaming and crying” for the past 45 minutes and that she cannot calm him down. She tells you that he did not lose consciousness. You advise her to bring him into the office, even though you just finished seeing your last patient and were just planning on leaving for the evening. When they arrive, you notice that the boy’s eyes appear very red from crying, but that he has calmed down. The mother is still very concerned. They have both been patients of yours for many years, and they have always been very healthy and compliant. You notice that there is a 0.5-cm edematous area on the back of his head. The skin is intact over the wound. The remainder of the physical examination, including a complete neurologic and funduscopic examination, is unremarkable. After calming the mother down, the most appropriate next step is to

  A. advise them to go to the emergency department for observation
  B. obtain a skull radiograph
  C. order a CT scan of the head
  D. recommend regular monitoring and observation for any abnormalities, and if they arise, that they should go to the emergency department
  E. report the case to the child protective service agency
Explanation:

The correct answer is D. This boy has a minor closed head injury with no loss of consciousness, which is one of the most common injuries in children. All children “bump” their heads at some point. It is important to do a physical examination, including a complete neurologic and funduscopic examination, and if this is normal, be able to recognize that further studies are generally not indicated. Regular monitoring and observation for any abnormalities can be done by a competent caregiver. If this occurred during the day, you may consider having them stay in your waiting room for a little while, but since you are leaving for the night and the mother has always been responsible and compliant, you can send them home for observation.

It is inappropriate to advise them to go to the emergency department for observation (choice A). This child had a minor head injury with no loss of consciousness and he has a normal neurologic and funduscopic examination. It is very unlikely that he has an intracranial injury, and therefore, as long as you know that the mother is a competent caretaker, you should send them home after advising her to seek assistance if he begins to deteriorate.

It is unnecessary to obtain a skull radiograph (choice B) at this time in this child with a minor closed head injury without a loss of consciousness. He does not have any signs of a skull fracture, which include battle signs (ecchymoses behind the ear), a palpable depression, or blood in the ear and therefore it is very unlikely that the radiograph will show a skull fracture. Also, even if he did have a skull fracture that does not necessarily mean that he has an intracranial injury.

A CT scan (choice C) is not indicated at this time in this patient with a minor closed head injury, no loss of consciousness, and a normal neurologic and funduscopic examination. Studies have shown that the risk of intracranial injury is negligible in this situation, and that fewer than 1 in 5,000 patients with minor closed head injuries and no loss of consciousness have intracranial injuries that require medical or neurosurgical intervention.

Since this case states that they are very healthy and compliant patients and the physical exam does not reveal any abnormalities besides this head wound that seems consistent with the story the mother told you, it is inappropriate to report the case to the child protective service agency (choice E). It is always important to be aware of signs of child abuse, such as many emergency room visits, many wounds in various stages of healing, implausible and inconsistent stories, and bringing the child in a while after the injury took place. This case does not seem to fit this description.

 

 

  A 31-year-old man is admitted to the hospital after a motorcycle accident. He was fully alert and oriented upon arrival and a primary survey revealed a left pneumothorax and small hemothorax with multiple rib fractures bilaterally. Other than some abrasions and lacerations requiring sutures, the patient was otherwise unremarkable. Initial laboratory results and electrocardiograms were unremarkable. After a chest tube was placed, the patient had reinflation of his left lung as demonstrated by chest radiograph. He is now concerned about further complications. The complication that this patient is at greatest risk for is

  A. aortic dissection
  B. empyema
  C. myocardial infarction
  D. pneumonia
  E. subarachnoid hemorrhage
Explanation:

The correct answer is D. Rib fractures are exquisitely painful. The magnitude of the pain forces people to halt their inspiratory efforts after small tidal volumes and prohibits them from effectively coughing and clearing secretions. This pattern of rapid, shallow breathing with no coughing predisposes patients to profound atelectasis of the lungs and infection due to residual secretions. Therefore he is at greatest risk for developing pneumonia. Often, these patients will receive rib blocks or thoracic epidural catheters for pain management. Opiates are not very effective since they produce somnolence, often worsening the respiratory problems.

All major chest trauma carries the possibility of aortic dissection (choice A), which is why chest radiographs are obtained to rule out a widened mediastinum. Although the negative predictive value of such films is not 100%, the fact that this patient has no chest or back pain coupled with a normal chest radiograph makes him especially low risk for this complication.

An empyema (choice B) is an infection of the pleural space. Since this patient has had his hemothorax drained, his risk of subsequent infection is essentially the same as the general population. His chest tube confers only a minimal increased risk of empyema.

With major myocardial contusions from blunt chest trauma, both direct myocardial injury as well as coronary dissection can result in infarction (choice C). This patient has a normal EKG, no symptoms of chest pain, and no evidence of major blunt chest trauma in the form of a hematoma or bruise across the sternum. For these reasons, his risk of infarction is quite low.

The most common cause of subarachnoid hemorrhage (choice E) is trauma. The presentation of SAH however is often directly after the trauma, and it is most often head trauma as the precipitating event. It is very rare for a SAH to have a delayed manifestation of hours or days after the trauma was suffered.

 

 

In early July, a 23-year-old female marathon runner is brought to the emergency department after collapsing during a 10 mile run. Her past medical history is significant for hypothyroidism and she is currently being treated with thyroid replacement hormones. On presentation she appears somnolent. Her skin is clammy and she is profusely diaphoretic. Her pulse is 110/min and blood pressure is 85/50 mm Hg. Her respirations are slow and shallow. Pulse oximetry measures 95% oxygen saturation on room air. A rectal thermometer shows a core body temperature of 40.8 C (105.4 F). Her lung sounds are clear. No murmurs were appreciated on cardiac auscultation. A neurologic examination shows dilated but reactive pupils and intact reflexes. There are noticeable petechiae over her trunk and right arm. Laboratory studies show:

A chest x-ray shows clear lungs and a normal cardiac silhouette. An electrocardiogram shows sinus tachycardia. Immediate cooling measures and an intravenous saline bolus are initiated. The patient is admitted to the intensive care unit for further management. Additional laboratory tests that are helpful in management of this condition are

  A. alanine aminotransferase, aspartate aminotransferase, alkaline phosphatase
  B. creatine kinase, urinalysis, fibrin split products
  C. lactate dehydrogenase, cardiac troponin, C-reactive protein
  D. serum albumin, erythrocyte sedimentation rate, anti-nuclear antibody
  E. thyroid stimulating hormone, thyroglobulin binding index, free thyroxine level
Explanation:

The correct answer is B. This patient is an athlete who presented to the emergency department with exertional heat stroke. Heat stroke can occur in athletes who exert themselves in conditions of high humidity and temperatures. Clinical signs of this condition include a core body temperature of >40.5 C (or 105 F), profuse sweating, and mental status changes. Patients with exertional heat stroke are at risk for disseminated intravascular coagulation and rhabdomyolysis. This patient’s presentation, prolonged coagulation times, evidence of petechiae, and low platelet count, is consistent with disseminated intravascular coagulation, and a positive test for fibrin split products will confirm the diagnosis. Secondly, heat stroke involves direct thermal damage to muscle tissue resulting in rhabdomyolysis. Muscle myoglobin deposition in renal tubules results in acute tubular necrosis and renal failure. A creatine kinase assay of >10, 000 UI/L will show skeletal muscle breakdown significant enough to cause acute renal failure. In addition, a urinalysis can detect the presence of urine myoglobin and monitor urine pH. Treatment for rhabdomyolysis involves intravenous saline infusions to maintain a high urine output and alkalization of urine with a target pH of 7-8 to prevent myoglobin deposition.

Alanine aminotransferase, aspartate aminotransferase, alkaline phosphatase (choice A) are predominantly liver function tests. Although they may be abnormal in this patient, they are not useful in the management of heat stroke.

Lactate dehydrogenase, cardiac troponin, and C-reactive protein (choice C) are useful in the diagnosis of myocardial infarction and do not aid in the therapy of heat stroke or rhabdomyolysis.

Serum albumin, erythrocyte sedimentation rate, and an anti-nuclear antibody test (choice D) are useful in evaluating a rheumatological process. They do not add information pertinent to the management of this patient.

The patient does have hypothyroidism, and these tests (choice E) would aid in treatment of her hypothyroidism. However, they are not helpful in treating her current condition.